SlideShare une entreprise Scribd logo
1  sur  34
Télécharger pour lire hors ligne
Ch Muhammad Irfan +92-345-4426176 facebook.com/chmuhammedirfan skype id:
ch.irfan786
Topic:
How do role model and support system influence entrepreneurs in their
career?
Role models -Individuals influencing an entrepreneur’s career choice and style
 Can be parents, family members, or other entrepreneurs.
 Successful entrepreneurs are viewed as catalysts by potential entrepreneurs.
 Role models can serve in a supportive capacity as mentors by providing information, advice
and guidance.
 Entrepreneurs need to establish connections and networks in the venture formation process
Moral-Support Network
It is important for the entrepreneur to establish a moral support network of family and friends.
Most entrepreneurs indicate that their spouses are their biggest supporters. Friends can provide
advice that is more honest than that received from others, plus encouragement, understanding,
and assistance. Relatives can also be sources of moral support, particularly if they are also
entrepreneurs.
Professional-Support Network
The entrepreneur also needs advice and counsel, which can be obtained from members of a
professional support network.
Topic:
List the characteristics that you believe are necessary for a successful
entrepreneur:
1. Disciplined
These individuals are focused on making their businesses work, and eliminate any hindrances or
distractions to their goals. They have overarching strategies and outline the tactics to accomplish
them. Successful entrepreneurs are disciplined enough to take steps every day toward the
achievement of their objectives.
2. Confidence
The entrepreneur does not ask questions about whether they can succeed or whether they are
worthy of success. They are confident with the knowledge that they will make their businesses
succeed. They exude that confidence in everything they do.
3. Open Minded
Entrepreneurs realize that every event and situation is a business opportunity. Ideas are
constantly being generated about workflows and efficiency, people skills and potential new
businesses. They have the ability to look at everything around them and focus it toward their
goals.
4. Self-Starter
Entrepreneurs know that if something needs to be done, they should start it themselves. They set
the parameters and make sure that projects follow that path. They are proactive, not waiting for
someone to give them permission.
5. Competitive
Ch Muhammad Irfan +92-345-4426176 facebook.com/chmuhammedirfan skype id:
ch.irfan786
Many companies are formed because an entrepreneur knows that they can do a job better than
another. They need to win at the sports they play and need to win at the businesses that they
create. An entrepreneur will highlight their own company’s track record of success.
6. Creativity
One facet of creativity is being able to make connections between seemingly unrelated events or
situations. Entrepreneurs often come up with solutions which are the synthesis of other items.
They will repurpose products to market them to new industries.
7. Determination
Entrepreneurs are not thwarted by their defeats. They look at defeat as an opportunity for
success. They are determined to make all of their endeavors succeed, so will try and try again
until it does. Successful entrepreneurs do not believe that something cannot be done.
8. Strong people skills
The entrepreneur has strong communication skills to sell the product and motivate employees.
Most successful entrepreneurs know how to motivate their employees so the business grows
overall. They are very good at highlighting the benefits of any situation and coaching others to
their success.
9. Strong work ethic
The successful entrepreneur will often be the first person to arrive at the office and the last one
to leave. They will come in on their days off to make sure that an outcome meets their
expectations. Their mind is constantly on their work, whether they are in or out of the workplace.
10. Passion
Passion is the most important trait of the successful entrepreneur. They genuinely love their
work. They are willing to put in those extra hours to make the business succeed because there is a
joy their business gives which goes beyond the money. The successful entrepreneur will always be
reading and researching ways to make the business better.
Topic:
Write a note on how entrepreneurs think:
 Entrepreneurs think by actions:
The entrepreneurial mindset desires action, so much so that they twist the age old saw to
read "Ready, Fire, Aim." Action for the entrepreneur tests their hypotheses and so provides them
with special market intelligence regardless of the outcome. Entrepreneurs think in possibilities:
Entrepreneurial thinking is not constrained by the reality of the present. Entrepreneurs see the
reality of possibilities that are not yet evident to others. As HBS professor Howard Stevenson
explains: "Entrepreneurs pursue opportunities without regard to resources currently
controlled." Entrepreneurs don't need all the resources in place before pursuing an opportunity ...
just the important ones.
 Entrepreneurs can think in ambiguity:
For the vast majority of people, ambiguity means confusion, disorganization and debilitation. Not
so for the entrepreneur who can still function effectively and move forward decisively in the
world of conflicting views, uncertain outcomes, and unresolved issues that exists as they craft and
hone their opportunity from the evolving mass of possibilities. The Lemonade entrepreneurial
Ch Muhammad Irfan +92-345-4426176 facebook.com/chmuhammedirfan skype id:
ch.irfan786
principle actually embraces surprises that arise from this uncertainty and will adjust goals on the
fly rather than be held by previous ones.
 Entrepreneurial thinking embraces risk:
While many money makers implement risk avoidance strategies to create wealth, the
entrepreneur knows that opportunities are more likely to exist for them by embracing and
managing risk rather than simply avoiding it. It is in these risky areas that the established players
avoid, that contains the gold nugget opportunities. Entrepreneurs have developed a whole host of
risk management treatments to help them in this regard like lean startups, customer
development process, beta launches, iterations and pivots, affordable loss experimentation,
'under the radar' launches, embedded entrepreneurship, and beachhead strategies. The
Affordable Loss Principle drives entrepreneurial action based on the acceptable downside, more
than it does on the attractiveness of the predicted upside.
 Entrepreneurs think in patterns:
Entrepreneurs are always looking for cause and effect patterns in the world around them.
Patterns help the entrepreneur to not only understand the world they live in but it also helps
them predict the future and to see opportunities that most people miss. These patterns may be
the ability to see the logical extension of a change or the creative-disruptive force of something
new or the cross pollination ability of an innovation into new fields. Entrepreneurs also know how
to apply the concepts and patterns from existing worlds to explain innovations in worlds as yet
unseen.
 Entrepreneurs think incessantly:
There is no off button in the entrepreneur's mind. They are incessantly curious, opportunistic, and
optimistic. They train their minds to problem-solve every issue that crosses their path. Every
person they meet is a potential resource, in every problem they encounter they seek a solution
and in every change they look for the new opportunity potential.
 Entrepreneurs think internally:
Rather than let others and external factors determine the value of outcomes, the entrepreneur
uses their internal locus of control to direct their destiny, to work for their achievements, to delay
gratification, and to plan with an eye for long-term benefits. Entrepreneurs never think of
themselves as victims. Everything they do has a personal development focus, even when doing
unappreciated work for others.
 Entrepreneurs think of value-add:
Rather than singularly employ a 'business-savvy' focus of extracting benefit by exploiting a
market/customer weakness, entrepreneurs focus primarily on creating new value for others (by
increased benefit or reduced cost) and then finding a way of securing a slice of that increased
value for themselves. This Crazy-Quilt entrepreneurial principle seeks partnerships with
committed people and organizations willing to jointly create the future and are least concerned
with traditional competitive analyses and strategic planning approaches.
 Entrepreneurs think of making meaning:
Ch Muhammad Irfan +92-345-4426176 facebook.com/chmuhammedirfan skype id:
ch.irfan786
Entrepreneurs have an inherent desire to make a difference and to make their world a better
place. So, for the entrepreneur, making money becomes a means to that end and not an end in
itself. As Richard Branson says: “Above all, you want to create something you are proud of ... I can
honestly say that I have never gone into any business purely to make money … I’ve had to create
companies that I believe in 100%. These are companies I feel will make a genuine difference.”
Topic:
Define entrepreneurship? Write a note on history of entrepreneurship.
The activity of setting up a business or businesses, taking on financial risks in the hope of profit.
The begging of Trade:
The original entrepreneurs were, of course, traders and merchants. The first known instance of
humans trading comes from New Guinea around 17,000 BCE, where locals exchanged obsidian, a
black volcanic glass used to make hunting arrowheads for other needed goods. These early
entrepreneurs exchanged one set of goods for another. Around 15,000 BCE, the first animal
domestication began taking place, and around 10,000 BCE, the first domestication of plants. This
step toward agriculture was critical for the advancement of the human species. Now, instead of
having to continually move around as nomadic tribes, seeking new places to hunt and to gather,
we could stay in one place.
The First Cities
The Middle East’s fertile crescent between the Tigris and the Euphrates had the right mix of plants
and animals to sustain the foundations of civilization. Around 4,000 BCE, people in central Asia
tamed horses, giving them a major advantage in both agricultural work and warfare. By 3,000
BCE, the first settlements and cities formed in Sumeria (modern day Iraq). During this timeframe,
the city of Uruk along the banks of the Euphrates River was home to 50,000 people in an amount
of space that would have previously supported just one hunter-gatherer. Humans had become
much more efficient at generating the food and energy necessary to support their communities.
Trade Routes Allow Ideas and Memes to Spread
Trade routes between the new cities soon sprang up. Donkeys, horses, and camels enabled trade
caravans between civilizations, moving both goods and ideas. Ships were built to carry trade over
the seas. Networks and hubs soon formed and more complex structures emerged. Great Pyramids
were built in Cairo. Temples were built in Sumeria.
The Invention of Money
Early trade consisted of barter (one good for another). If Tom had twenty cows and Igor had
eighty hens, and Tom and Igor agreed that one cow was worth four hens, then the trade could
take place. The problem with the barter system, however, was that in order for a trade to take
place, both parties had to want what the other party had. This “co-incidence of wants” often did
not happen. Thus, the demands of growing business and trade gave rise to a money system. Silver
rings or bars are thought to have been used as money in Ancient Iraq before 2000 B.C. Early forms
Ch Muhammad Irfan +92-345-4426176 facebook.com/chmuhammedirfan skype id:
ch.irfan786
of money (called specie) would be often be commodities like seashells, tobacco leaves, large
round rocks, or beads.
The Creation of Markets
With a population spurt starting around 1470, cities, markets, and the volume of trade grew.
Banking, initially started by Ancient Mesopotamians, grew to new heights and complexities; the
guild system expanded; and the idea that a business was an impersonal entity, with a separate
identity from its owner, started to take hold. Silver imports from the new world drove expanded
trade and bookkeepers created standardized principles for keeping track of a firm’s accounts
based on Luca Pacioli’s accounting advances. Early entrepreneurs, called merchants and
explorers, began to raise capital, take risks, and stimulate economic growth. Capitalism had
begun.
Define Entrepreneurship? Write a note on entrepreneurial process:
The Entrepreneur is a change agent that acts as an industrialist and undertakes the risk associated
with forming the business for commercial use. An entrepreneur has an unusual foresight to
identify the potential demand for the goods and services.
1. Discovery: An entrepreneurial process begins with the idea generation, wherein the
entrepreneur identifies and evaluates the business opportunities. The identification and
the evaluation of opportunities is a difficult task; an entrepreneur seeks inputs from all the
persons including employees, consumers, channel partners, technical people, etc. to reach
to an optimum business opportunity. Once the opportunity has been decided upon, the
next step is to evaluate it.
An entrepreneur can evaluate the efficiency of an opportunity by continuously asking certain
questions to himself, such as, whether the opportunity is worth investing in, is it sufficiently
attractive, are the proposed solutions feasible, is there any competitive advantage, what are the
risk associated with it. Above all, an entrepreneur must analyze his personal skills and hobbies,
whether these coincides with the entrepreneurial goals or not.
2. Developing a Business Plan: Once the opportunity is identified, an entrepreneur needs to
create a comprehensive business plan. A business plan is critical to the success of any new
venture since it acts as a benchmark and the evaluation criteria to see if the organization is
moving towards its set goals.
An entrepreneur must dedicate his sufficient time towards its creation, the major components of
a business plan are mission and vision statement, goals and objectives, capital requirement, a
description of products and services, etc.
3. Resourcing: The third step in the entrepreneurial process is resourcing, wherein the
entrepreneur identifies the sources from where the finance and the human resource can
be arranged. Here, the entrepreneur finds the investors for its new venture and the
personnel to carry out the business activities.
4. Managing the company: Once the funds are raised and the employees are hired, the next
step is to initiate the business operations to achieve the set goals. First of all, an
Ch Muhammad Irfan +92-345-4426176 facebook.com/chmuhammedirfan skype id:
ch.irfan786
entrepreneur must decide the management structure or the hierarchy that is required to
solve the operational problems when they arise.
5. Harvesting: The final step in the entrepreneurial process is harvesting wherein, an
entrepreneur decides on the future prospects of the business, i.e. its growth and
development. Here, the actual growth is compared against the planned growth and then
the decision regarding the stability or the expansion of business operations is undertaken
accordingly, by an entrepreneur.
The entrepreneurial process is to be followed, again and again, whenever any new venture is
taken up by an entrepreneur, therefore, it’s an ever ending process.
Topic:
As an entrepreneur how do you establish a Corporate Entrepreneurship in the
organization:
Corporate entrepreneurship is especially crucial for large companies, enabling these organizations
- that are traditionally averse to risk-taking - to innovate, driving leaders and teams toward an
increased level of corporate enterprising. In addition to the obvious benefits obtained through
innovation, this approach also provides the organizational benefit of setting the stage for
leadership continuity.
Step one:
Secure a commitment to corporate entrepreneurship in the organization by top, upper, and
middle management levels. Establish initial framework and embrace the concept identify, select,
and train
Step two:
Identify ideas and areas that top management is interested in supporting. Identify amount of risk
money available to develop the concept. Establish overall program expectations and target results
of each corporate venture. Establish mentor/sponsor system.
Step three:
Use of technology to ensure organizational flexibility. corporate entrepreneurs.
Step four:
Identify interested managers to train employees and share their experiences.
Step five:
Develop ways for the organization to get closer to its customers.
Step six:
Learn to be more productive with fewer resources.
Step seven:
Establish a strong support structure for corporate entrepreneurship.
Step eight:
Tie rewards to the performance of the entrepreneurial unit.
Finally:
Implement an evaluation system that allows successful entrepreneurial units to expand and
unsuccessful ones to be eliminated.
Ch Muhammad Irfan +92-345-4426176 facebook.com/chmuhammedirfan skype id:
ch.irfan786
Topic:
Discuss the various difference b/w managerial and entrepreneurial decision
making:
The difference between the entrepreneurial style and the managerial style (administrative
domain) involves five business dimensions.
A. Strategic Orientation.
1. The entrepreneur’s strategic orientation depends on his or her perception of the opportunity.
2. When the use of planning systems is the strategic orientation, the administrative domain is
operant.
C. Commitment to Opportunity.
1. The entrepreneurial domain is pressured by the need for action and has a short time span in
terms of opportunity commitment.
2. The administrative domain is not only slow to act on an opportunity, but the commitment is
usually for a longer time span.
D. Commitment of Resources.
1. An entrepreneur is used to having resources committed at periodic intervals, often based on
certain tasks or objectives being reached.
2. In acquiring these resources the entrepreneur is forced to maximize resource use.
3. In the administrative domain, the commitment of resources is for the total amount needed.
4. Administrative-oriented individuals receive personal rewards by effectively administering the
resources under their control.
E. Control of Resources.
1. The administrator is rewarded by effective resource administration and has a drive to own or
accumulate as many resources as possible.
2. The entrepreneur, under pressure of limited resources, strives to rent resources on an as-
needed basis.
F. Managerial Structure.
1. In the administrative domain, the organizational structure is formalized and hierarchical in
nature.
2. The entrepreneur employs a flat organizational structure with informal networks.
Topic:
Define new entry. How is a new entry opportunity generated?
Ch Muhammad Irfan +92-345-4426176 facebook.com/chmuhammedirfan skype id:
ch.irfan786
Offering a new product to an established or new market.
 Resources as a Source of Competitive Advantage
Resources are the basic building blocks to a firm’s functioning and performance; the inputs into
the production process.
i. They can be combined in different ways.
ii. A bundle of resources provides a firm its capacity to achieve superior performance.
Resources must be:
i. Valuable.
ii. Rare.
iii. Unique.
 Creating a Resource Bundle That Is Valuable, Rare, and Inimitable
i. Entrepreneurs need to draw from their unique experiences and knowledge.
ii. Market knowledge - Information, technology, know-how, and skills that provide insight
into a market and its customers.
iii. Technological knowledge - Information, technology, know-how, and skills that provide
insight into ways to create new knowledge.
 Assessing the Attractiveness of a New Entry Opportunity
Depends on the level of information and the willingness to make a decision without perfect
information.
 Information on a New Entry
i. Prior knowledge and information search
ii. More knowledge ensures a more efficient search process.
iii. Search costs include time and money.
iv. The viability of a new entry can be described in terms of a window of opportunity.
 Comfort with Making a Decision under Uncertainty
i. The trade-off between more information and the likelihood that the window of
opportunity will close provides a dilemma for entrepreneurs.
ii. Error of commission - Negative outcome from acting on the perceived opportunity.
iii. Error of omission - Negative outcome from not acting on the new entry opportunity.
Topic:
Discuss the various risk reduction strategies:
Select a business structure that limits personal liability.
Change your business structure from a sole proprietorship in which you are personally liable for
business operations to a corporation or limited liability company where you have limited liability.
Transfer risk to insurance companies by insuring against major risks
Transfer risk to insurance companies by insuring against major risks such as damage to your
facilities, product liability, injuries to customers or suppliers and death or incapacity of company
principals.
Perform a risk analysis by evaluating the consequences of risky activities
Ch Muhammad Irfan +92-345-4426176 facebook.com/chmuhammedirfan skype id:
ch.irfan786
Perform a risk analysis by evaluating the consequences of risky activities, the likelihood of the
consequences occurring and the benefits of the risky activities. Avoid risk by not carrying out
activities that have severe and likely consequences and low benefits.
Transfer the risk of activities
Transfer the risk of activities with severe and likely consequences but high benefits to other
parties. Create a new, independent company to carry out these activities or assign them to
suppliers or partners.
Reduce risk from product failure and warranty claims
Reduce risk from product failure and warranty claims by implementing a quality assurance
program. Develop a system of reporting from customer service to identify problems. Structure the
quality assurance program to document production tasks and product testing. Link the problems
reported by customer service to specific failures in production or testing procedures and institute
corrective action.
Reduce risk of surprises in operating results
Reduce risk of surprises in operating results by keeping accurate records and instituting effective
controls. Put in place a system that limits who can authorize specific actions and how much they
can spend. Implement a reporting system that gives you key information about company
performance. Evaluate the controls and reporting system by comparing actual practice and
performance to the control procedures and the reported information.
Reduce financial risk by managing your accounts receivable
Reduce financial risk by managing your accounts receivable to minimize outstanding balances and
identify poor credit risks. Implement credit and payment standards, specifying which credit scores
and payment records are acceptable. Evaluate customer payments and ask for advance payment
from customers who don't meet the standards.
Reduce financial risk by keeping outstanding loans and financing needs to a
minimum
Control growth at a rate that the company can finance internally. If the company can't pay off
some loans, replace short-term credit with long-term, fixed-rate loans.
Topic:
Explain the importance of entrepreneurial background and characteristics:
ENTREPRENEURIAL BACKGROUND
Only a few background characteristics have differentiated the entrepreneur from the general
populace or managers.
Childhood Family Environment
The impact of birth order and social status has had conflicting research results. There is strong
evidence that entrepreneurs, both male and female, tend to have self-employed or
entrepreneurial fathers. Having a father who is self-employed provides a strong inspiration in the
example of independence and flexibility of self-employment. This feeling of independence is often
further enforced by an entrepreneurial mother.
Education
Ch Muhammad Irfan +92-345-4426176 facebook.com/chmuhammedirfan skype id:
ch.irfan786
1. Education appears important in the upbringing of the entrepreneur, in the level of education
obtained and in playing a major role in coping with problems.
2. Although formal education is not necessary for starting a new business, it does provide a good
background.
3. In education, entrepreneurs experienced some disadvantage, with few having degrees in
engineering, science, or math.
4. The ability to deal with people and communicate clearly in written and spoken work is also
important.
Age
 Entrepreneurial age is the age of the entrepreneur reflected in the experience.
 Entrepreneurial experience is one of the best predictors of success.
 In chronological age, most entrepreneurs start their careers between ages 22 and 55.
 Earlier starts in an entrepreneurial career seem to be better than later ones.
 Generally, male entrepreneurs start their first venture in their early 30s, while women tend to
do so in their middle 30s.
 Between 30-50 years’ old
Work History
1. Dissatisfaction with one’s job often motivates the launching of a new venture.
2. (Gift within trouble)
3. Previous technical and industry experience is also important once the decision to start a
business is made.
4. Experience in the following areas is particularly important: financing; product or service
development; manufacturing; development of distribution channels; and preparation of a
marketing plan.
5. As the venture becomes established, managerial experience and skills become more
important.
6. Entrepreneurial experience becomes increasingly important as the complexity of the
venture increases.
MOTIVATION
1) While motivations may vary, the reason cited most often for becoming an entrepreneur
independence-not wants to work for anyone else.
2) Other motivating factors differ between male and female entrepreneurs.
3) Money is the second reason for men’s starting a venture.
4) Job satisfaction, achievement, opportunity, and money are the second order reasons for
women.
Entrepreneurial Characteristics:
1. Key Personal Attributes
2. Strong Managerial Competencies
3. Good Technical Skills
Topic:
Identify and define the methods of Generating new ideas:
Ch Muhammad Irfan +92-345-4426176 facebook.com/chmuhammedirfan skype id:
ch.irfan786
Focus Groups
A focus group is defined as a group of individuals providing information in a structured format. A
moderator leads the group of about 8 to 14 participants through an open, in-depth discussion
rather than simply asking questions to solicit participants’ responses.
For a new product or services area, the moderator focuses the discussion of the group in either a
directive or non-directive manner.
The group is stimulated by comments from other group members in creatively conceptualizing
and developing a new product or service to fill a market need.
The focus group is an excellent approach for initially screening ideas and concepts. Existing
company can use this method to expand a section or department to be able to achieve greater
productivity in its services.
Brainstorming
The brainstorming method allows a group of individual to be stimulated to greater creativity by
interacting with each other and participating in organized group experience.
Most of the ideas generated from the group have no basis for further development, yet there are
times that a good idea emerges. This has a greater frequency of occurrence when the
brainstorming effort focuses on a specific product or market area.
For this approach to be successful there should be no negative comment or criticism, quantity of
ideas should be encouraged, the wider the idea, the better and combinations and improvements
of ideas are encouraged.
Many modern commercial banks successfully used brainstorming techniques to develop a series
of product for their clients or customers.
Problem Inventory Analysis
Problem inventory analysis is a method for obtaining new ideas and solutions by focusing on
existing problems. In this approach, the customers or consumers are provided with a list of
problems in a general product category.
Thereafter, they are asked to identify and discuss product in each category that have a particular
problem. This method is effective when an improved service/product is desired.
When known product or services are related to suggested problems, a new product idea emerges.
Result from product inventory analysis must be carefully evaluated as they may actually reflect a
new business opportunity.
For maximal result, it is advisable that problem inventory analysis should be used primarily to
identify new product ideas from existing product before further evaluation.
Topic:
Discuss the seven creative problems solving techniques:
Find the Right Problems to Solve
Too often our approach to problem solving is reactive, we wait for the problems to arise. Firstly,
in our 7 problem solving steps, we advocate taking a proactive approach, go and find problems to
solve; important and valuable problems. The real starting point then for any problem solving
process is to find the right problem to solve.
 Define the Problem
Ch Muhammad Irfan +92-345-4426176 facebook.com/chmuhammedirfan skype id:
ch.irfan786
It is very tempting to gloss over this step and move to analysis and solutions. However, like
the first step, it is one of the secrets of effective problem solving. Combining problems that
are valuable to solve, with defining exactly what you are trying to solve, can dramatically
improve the effectiveness of the problem solving process. The secret to defining the
problem, is really about attitude. Try to see every problem as an opportunity.
 Analyze the Problem
Analysis is a process of discovery of the facts, finding out what you know about the
situation. In doing so you are breaking down complexity, stripping away the superficial and
getting to the causes/issues.
 Develop Opportunities
There are always more than one way to solve a problem, so take time to develop plenty of
creative possibilities to solve the problem.
 Select the Best Solution
Selecting is about making choices, about deciding. To do this you need to weigh up the
competing value and risk of the different options you generated in the previous step.
 Implement
Good solutions are often only as good as the way they are implemented. Implementation
requires project management and a determination to deliver the outcomes essential to
solving the problem you originally defined.
 Evaluate and Learn
You will have done some things really well through this seven step problem solving
process. It would be all too easy to forget them in rushing to solve the next problem, or to
implement the solution. You should evaluate at least two areas:
How you carried out the seven step problem solving process
The effectiveness of the solution you implemented. Did it deliver the outcomes you
expected?
Topic:
Differentiate between domestics and international entrepreneurship?
International Entrepreneurship:
Global entrepreneurship is a trait or characteristic in our definition of a global leader. Global
entrepreneurs are professionals who use their global understanding and connections to identify
transnational and cross-cultural opportunities and turn them into new value-creating initiative.
Doing business internationally is not the same as doing business at home. There are new skills to
learn and new knowledge to acquire about the country you will be going into. You will need to
learn about the different laws and regulations, the different customer buying habits, and change
your marketing strategies and materials to appeal to the new country you are entering.
Cultures
No two cultures are the same and understanding both the social and business culture in another
country is the first key to success. Culture defines everything a society does, from its business
Ch Muhammad Irfan +92-345-4426176 facebook.com/chmuhammedirfan skype id:
ch.irfan786
practices, to its response to advertising and marketing, to negotiating sales. It is important to
include research on the culture of the country(s) that you intend to sell to prior to entering their
market. Understanding these, often sensitive, areas will mean that you are better prepared when
first entering the market.
Level of Competition
The level of competition you will experience in foreign markets is likely to be more dynamic and
complex than you experience in domestic markets.
Market Intelligence
The key points to determine when gathering market intelligence on the market you intend to
enter are:
 Understanding how the market works
 Who your direct competition is, and
 The best market entry strategy
Politics/Government/Legal Systems
No two countries have the same political and legal systems. Each government has its own policies
relating to foreign firms and products. The key is to understand that once you are in a foreign
market you must abide by the rules and laws of that country, not the ones in your own market.
International Law
Countries determine their laws based on the needs of their citizens not the concerns of foreign
companies. By and large, international law is a gentlemen's agreement which is honored, but not
always.
Conclusion:
There are a many differences between domestic and international business. Whether they are
cultural, technical or legal they require an understanding and an appreciation of the differences.
Following are a few web links to help you get started.
Topic:
Explain the sources of new product development ideas for entrepreneurs
with examples:
 Internal Sources:
Internal sources are the great way to find new ideas. Internal sources can be divided into
two parts. They are as follows-
1. R & D (Research and Development): It is the formal department of any organization to
generate new ideas. R&D department research according to the company’s future plan and
then come up with the new ideas which complete its journey with the commercialization
of the idea (product). One recent survey showed that traditional R&D only contribute less
than 15% of the ideas of the organization. However, the importance of R&D in the
organization is not only depends on the idea generation, they also do some other
important things.
2. Employees: Companies can use the brain of their employees. If customers are
the Oxygen of any company, then employees are the heart of that company. All level of
employees from executive to top management can be the great source of ideas. One
Ch Muhammad Irfan +92-345-4426176 facebook.com/chmuhammedirfan skype id:
ch.irfan786
recent research showed that almost 45% of the ideas comes from the employees. Many
companies now use web technology to get the ideas from their internal employees. In that
web form employees can share their ideas about new product. However, picking up the
great idea from it and rewarding the employees can encourage your employees to be more
creative and contribute more in future.
 External Sources:
Companies can also gain good new product development ideas from external sources. External
sources are those which is affiliated with the company externally. Some important external
sources for idea generation is discussing below.
3. Customers: Most probably customers are the most important sources to get new product
development ideas. Customer knows best what they need and what they are looking for. It
is the most important thing to deliver satisfaction by providing exactly what your
customers want. For instances, when you know that your customer need a specific product
or a special feature on any particular product then it will be easier to make that exactly
what your customer need and then you will get satisfied customers. Thus way you can
build long term relationship.
4. Distributors and Suppliers: Distributors works very closely with the market and they know
consumer problems and their need. Distributors can give the ideas for new product
possibilities. Suppliers can also help with the information of the market like new concept,
technique or materials which can be used for developing new products.
5. Competitors: Competitors are another important source. One can analyze their
competitors and can find many things which can be used for idea generation. Researching
competitors can give you the idea that which thing you are missing or which thing they are
missing, you can decide then what things you need to include in your new product.
Remember your competitors are not your enemy, they are your strength.
6. Others: Other idea sources includes outside Consultancies, Design Firms and Online
Communities, Trade Magazines, Shows and Seminars, Government agencies, Advertising
agencies, Marketing research firms, Universities, Commercial laboratories, Inventories and
so on.
Topic:
Write a note on product planning and development process:
New Product Development:
Developing a new product involves a number of stages which typically center on the following key areas:
 The Idea:
Every product has to start with an idea. In some cases, this might be fairly simple, basing the new product
on something similar that already exists. In other cases, it may be something revolutionary and unique,
which may mean the idea generation part of the process is much more involved. In fact, many of the
leading manufacturers will have whole departments that focus solely on the task of coming up with ‘the
next big thing’.
 Research:
Ch Muhammad Irfan +92-345-4426176 facebook.com/chmuhammedirfan skype id:
ch.irfan786
An organization may have plenty of ideas for a new product, but once it has selected the best of them, the
next step is to start researching the market. This enables them to see if there’s likely to be a demand for
this type of product, and also what specific features need to be developed in order to best meet the
needs of this potential market.
 Development:
The next stage is the development of the product. Prototypes may be modified through various design
and manufacturing stages in order to come up with a finished product that consumers will want to buy.
 Testing:
Before most products are launched and the manufacturer spends a large amount of money on production
and promotion, most companies will test their new product with a small group of actual consumers. This
helps to make sure that they have a viable product that will be profitable, and that there are no changes
that need to be made before it’s launched.
 Analysis:
Looking at the feedback from consumer testing enables the manufacturer to make any necessary changes
to the product, and also decide how they are going to launch it to the market. With information from real
consumers, they will be able to make a number of strategic decisions that will be crucial to the product’s
success, including what price to sell at and how the product will be marketed.
 Introduction:
Finally, when a product has made it all the way through the new product development stage, the only
thing left to do is introduce it to the market. Once this is done, good product life cycle management will
ensure the manufacturer makes the most of all their effort and investment.
Topic:
Identify and define the various Entry Modes of international entrepreneurs:
The decision of how to enter a foreign market can have a significant impact on the results.
Expansion into foreign markets can be achieved via the following four mechanisms:
Exporting
Exporting is the marketing and direct sale of domestically produced goods in another country.
Exporting is a traditional and well established method of reaching foreign markets. Since
exporting does not require that the goods be produced in the target country, no investment in
foreign production facilities is required. Most of the costs associated with exporting take the form
of marketing expenses.
 Licensing
Ch Muhammad Irfan +92-345-4426176 facebook.com/chmuhammedirfan skype id:
ch.irfan786
Licensing essentially permits a company in the target country to use the property of the licensor.
Such property usually is intangible, such as trademarks, patents, and production techniques. The
licensee pays a fee in exchange for the rights to use the intangible property and possibly for
technical assistance.
Joint Venture
There are five common objectives in a joint venture: market entry, risk/reward sharing,
technology sharing and joint product development, and conforming to government regulations.
Other benefits include political connections and distribution channel access that may depend on
relationships.
Foreign Direct Investment
Foreign direct investment (FDI) is the direct ownership of facilities in the target country. It
involves the transfer of resources including capital, technology, and personnel. Direct foreign
investment may be made through the acquisition of an existing entity or the establishment of a
new enterprise.
Topic:
What are the steps in registering your trademark of your business?
Procedure/Steps for Trademark Registration
 Filing of an application for registration by a person claiming to be the proprietor of a
trademark, in the office of the Trade Mark Registry, within the territorial limits of the place
of business in India.
 Examination of the application by the Registrar to ascertain whether it is distinctive and
does not conflict with existing registered or pending trademarks and examination report is
issued.
 Publication of the application after or before acceptance of the application in the
Trademark Journal.
 After publication if any person gives notice of his opposition to the registration within
three months which may be extended to the maximum of one month.
 If the opposition has been decided in favor of the applicant of the registration of
trademark, the Registrar shall register the Trademark.
 On the registration of the Trademark the Registrar shall issue to the applicant a Trademark
Registration Certificate.
Topic:
What is business plan? Write down all business segment for a business plan:
Definition:
A written document describing the nature of the business, the sales and marketing strategy, and
the financial background, and containing a projected profit and loss statement.
 Title Page and Contents
A business plan should be presented in a binder with a cover listing the name of the
business, the name(s) of the principal(s), address, phone number, e-mail and website
Ch Muhammad Irfan +92-345-4426176 facebook.com/chmuhammedirfan skype id:
ch.irfan786
addresses, and the date. You don't have to spend a lot of money on a fancy binder or
cover. Your readers want a plan that looks professional, is easy to read and is well-put-
together.
 Executive Summary
The executive summary, or statement of purpose, succinctly encapsulates your reason for
writing the business plan. It tells the reader what you want and why, right up front. Are you
looking for a $10,000 loan to remodel and refurbish your factory? A loan of $25,000 to
expand your product line or buy new equipment? How will you repay your loan, and over
what term? Would you like to find a partner to whom you'd sell 25 percent of the
business? What's in it for him or her? The questions that pertain to your situation should
be addressed here clearly and succinctly.
Description of the Business
The business description usually begins with a short explanation of the industry. When describing
the industry, discuss what's going on now as well as the outlook for the future. Do the necessary
research so you can provide information on all the various markets within the industry, including
references to new products or developments that could benefit or hinder your business. Base
your observations on reliable data and be sure to footnote and cite your sources of information
when necessary. Remember that bankers and investors want to know hard facts--they won't risk
money on assumptions or conjecture.
Description of the Product or Service
The business description can be a few paragraphs to a few pages in length, depending on the
complexity of your plan. If your plan isn't too complicated, keep your business description short,
describing the industry in one paragraph, the product in another, and the business and its success
factors in two or three more paragraphs.
 Market Analysis
A thorough market analysis will help you define your prospects as well as help you
establish pricing, distribution, and promotional strategies that will allow your company to
be successful vis-à-vis your competition, both in the short and long term.
 Competitive Analysis
The purpose of the competitive analysis is to determine:
 the strengths and weaknesses of the competitors within your market.
 strategies that will provide you with a distinct advantage.
 barriers that can be developed to prevent competition from entering your market.
 any weaknesses that can be exploited in the product development cycle.
 Operations and Management
The operations and management component of your plan is designed to describe how the
business functions on a continuing basis. The operations plan highlights the logistics of the
organization, such as the responsibilities of the management team, the tasks assigned to
each division within the company, and capital and expense requirements related to the
operations of the business.
Ch Muhammad Irfan +92-345-4426176 facebook.com/chmuhammedirfan skype id:
ch.irfan786
 Financial Components of Your Business Plan
After defining the product, market and operations, the next area to turn your attention to
are the three financial statements that form the backbone of your business plan: the
income statement, cash flow statement, and balance sheet.
Topic:
Write a note on Followings:
Patents:
Patent protect functional expressions of an idea – not the idea itself. A machines,
method/process, manufacture, compositions of matter, and improvements of any of these items
can be patented. Thus, I can patent a design for the nozzle on a rocket, or the method of making
the rocket, or the method of making the rocket fuel, or the metal in which the rocket fuel is
stored, or a new way of transporting the rocket fuel to the rocket. But I cannot patent the broad
“idea” of a rocket.
Copyrights:
Copy Rights protect the specific creative expression of an idea through any medium of
artistic/creative expression – i.e., paintings, photographs, sculpture, writings, software, etc. A
copyright protects your painting of a haystack, but it would not prohibit another painter from
expressing their artistry or viewpoint by also painting a haystack. Likewise, while Ian Fleming was
able to receive a copyright on his particular expression of the idea of a secret agent (i.e., a
debonair English secret agent), he could not prevent Rich Wilkes from receiving a copyright on his
expression of the idea of a secret agent (i.e., a tattooed bald extreme athlete turned reluctant
secret agent).
Trademarks:
Trade marks protect any symbol that indicates the source or origin of the goods or services to
which it is affixed. While a trademark can be extremely valuable to its owner, the ultimate
purpose of a trademark is to protect consumers – that is, the function of a trademark is to inform
the consumer where the goods or services originate. The consumer, knowing the origin of the
goods, can make purchasing decisions based on prior knowledge, reputation or marketing.
Trade Secrets:
Broadly speaking, any confidential business information which provides an enterprise a
competitive edge may be considered a trade secret. Trade secrets encompass manufacturing or
industrial secrets and commercial secrets. The unauthorized use of such information by persons
other than the holder is regarded as an unfair practice and a violation of the trade secret.
Depending on the legal system, the protection of trade secrets forms part of the general concept
of protection against unfair competition or is based on specific provisions or case law on the
protection of confidential information.
Topic:
Write down the outline of your business plan:
Executive summary
Ch Muhammad Irfan +92-345-4426176 facebook.com/chmuhammedirfan skype id:
ch.irfan786
Begin your plan with an executive summary, which you should write last. The summary condenses
your main points into two or three pages so investors can quickly decide if the opportunity suits
them enough to keep reading.
Product
What are you selling? Describe your product, giving enough information to imagine the product.
Eliminate the technical details-if you can't get the idea across quickly, you haven't thought it
through enough.
Marketing
Who buys, why and how? Who are your customers? What need are you filling? Give numbers and
sources to give a feel for the opportunity size. For example, "Tractors Quarterly, May 2000, says
20,000 tractor manufacturers need 15 hours to change production lines. Our patent-protected
TractorGizmoT reduces that to 12 seconds, saving an estimated $55 billion yearly."
Business model
Show where the money comes from. Gillette loses money on razors, but makes money-lots of
money-selling blades. Discuss who pays you, how much they pay and how often
The team
Introduce your team and advisory board. Good ideas are common; talented managers aren't.
Show you have the right team for the idea by highlighting the relevant background of your team
members. If you make ball bearings, emphasize your experience in manufacturing, not your time
as a college newspaper editor.
Competition
What's your competition and why will you win? Competition may not be a company. Personal
checkbook program Quicken considered pencil and paper, not other computer programs, the
biggest competition.
Financing
How much money do you have? Where did it come from? How much do you want? How will you
use it? What return are you promising investors?
Operations
Eventually you'll need to get things done. Describe how you'll deliver your product, the problems
you expect and how you'll master them. If your business allows 100,000 people a year to order
custom-made suits by telephone, here's where you say how you'll actually make and deliver
100,000 suits.
Scale
How will you grow? A diner isn't run the same way as a 250-table restaurant. Explain your growth
plans and how your idea scales.
Development
How are you doing so far? If you're already under way, talk about your progress. In any event, lay
out the next few months' milestones so you and your investors can track your progress.
Topic:
Ch Muhammad Irfan +92-345-4426176 facebook.com/chmuhammedirfan skype id:
ch.irfan786
List all methods of generating new ideas for the entrepreneurial venture and
explain any three of them:
METHODS OF GENERATING NEW IDEAS
Even with the wide variety of sources available, coming up with an idea to serve as the basis for
the new venture can still be a difficult problem. The entrepreneur can use several methods to
help generate and test new ideas, including focus groups, brain storming and problem inventory
analysis.
Focus groups
Group of individuals providing information in a structured format is called a focus group. The
group of 8 to 14 participants is simulated by comments from other group members in creatively
conceptualizing and developing new product idea to fulfill a market need.
Brainstorming
A group method of obtaining new ideas and solutions is called brainstorming. The brainstorming
method for generating new ideas is based on the fact that people can be stimulated to greater
creativity by meeting with others an d participating with organized group experiences. Although
most of the ideas generated from the group have no basis for further development, often a good
idea emerges.
Problem inventory analysis
Problem inventory analysis uses individuals in a manner that is analogous to focus groups to
generate new product ideas. However instead of generating new ideas themselves, consumers
are provided with a list of problems in a general product category. They are then asked to identify
and discuss products in this category that have the particular problem. This method is often
effective since it is easier to relate known products to suggested problems and arrive at a new
product idea then to generate an entirely new idea by itself.
Topic:
What are the characteristics of a Marketing plan?
You have a product people love and you understand who the people are that love it. Now would
be a great time to build a marketing plan. The plan will change and evolve over time but having a
structure helps to make sure that your marketing spend is aligning with your goals without being
completely ad-hoc, spray and pray, or flavor of the day. The plan shouldn’t be a massive
undertaking – it’s a rough guide that documents what you are planning on doing and why you are
doing it. Good plans should contain the following:
Segmentation –
At this stage you should understand what segments love you offering and why. If you don’t, you
probably aren’t ready to build a marketing plan yet and you should go back and focus on finding a
fit between your offering and a set of markets. This is where you document with as much detail
as humanly possible the segments that you are going to target. Who are the groups, what are
their characteristics and how do you identify them?
Ch Muhammad Irfan +92-345-4426176 facebook.com/chmuhammedirfan skype id:
ch.irfan786
Competitive Alternatives –
This is a documented list of what your customers would consider alternatives to your product or
service. Don’t go crazy on this documenting every little bit of functionality. The purpose of this
section is to help you articulate what really differentiates your offering from others in the
space. These are generally macro things and not niggly detailed nuances of features.
Differentiated Points of Value (by segment) –
For each segment, what are the top 3 or 4 differentiators that your offering has versus
others. Remember this isn’t just about technology or features. It often includes things like
pricing, delivery options, ease of use, time to value, etc.
Messaging and Positioning (by segment) –
Working from the previous section create a set of messages for each segment. This should be a
succinct set of no more than 3/4 messages that get across your key points. (For more on this see
my post on Crafting Simple Value Statements)
Marketing Goals and Measures –
What are the goals of your marketing plan and what metrics are you tracking that are associated
with those goals? For example, you might decide that increasing customer acquisition by x% is a
key goal. You can tie acquisition to site visits, product signups, email or blog signups, abandon
rates, etc.
Tactical Plan, Budget, Owners –
Based on the above goals and measures this is the set of marketing plans you plan on executing to
drive those results and the costs associated with each of those tactics. The tactics are broken into
discreet items of work (i.e. for example a tactic such as a webinar will include creating the invite
list, writing/designing the mailer, sending the invite, creating the webinar content, etc.) and
assigned to an owner.
Timeline–
The tactics need to be broken into work items and plotted on a timeline so they can be tracked on
a regular (for my teams it’s usually weekly) basis.
OR
Eight parameters can help in gauging the viability of strategic marketing plans. If your planning
process reflects these attributes, it's probably off to a good start.
 Accountability—responsibility is assigned for successful completion of initiatives.
 Balance—the plans inform and guide not only financial decision-making, but also
operational and human resource issues.
 Flexibility—a mechanism for changing and updating the plan is built into the process.
Ch Muhammad Irfan +92-345-4426176 facebook.com/chmuhammedirfan skype id:
ch.irfan786
 Manageability—in-process measures are identified to ensure processes are working as
intended, critical performance issues are addressed, resources required are projected, and
methods of status reporting are in place.
 Prioritization—priorities are established whenever multiple interdependent actions have
been planned.
 Realism—the question of what the organization really can do versus what it would like to
do is addressed rationally, though the tone may be optimistic.
 Specificity—expected results and milestones are clearly defined, along with the specific
actions for implementation and the deliverables for each step.
 Sustainability—a sufficient time period is allowed to close performance gaps.
Topic:
Write a note on steps in marketing research:
Step 1. Define the Objective & Your “Problem”
Perhaps the most important step in the market research process is defining the goals of the
project. At the core of this is understanding the root question that needs to be informed by
market research. There is typically a key business problem (or opportunity) that needs to be
acted upon, but there is a lack of information to make that decision comfortably; the job of a
market researcher is to inform that decision with solid data.
Step 2. Determine Your “Research Design”
Now that you know your research object, it is time to plan out the type of research that will best
obtain the necessary data. Think of the “research design” as your detailed plan of attack. In this
step you will first determine your market research method (will it be a survey, focus group,
etc.?). You will also think through specifics about how you will identify and choose your sample
(who are we going after? where will we find them? how will we incentivize them? etc.).
Step 3. Design & Prepare Your “Research Instrument”
In this step of the market research process, it’s time to design your research tool. If a survey is
the most appropriate tool (as determined in step 2), you’ll begin by writing your questions and
designing your questionnaire. If a focus group is your instrument of choice, you’ll start preparing
questions and materials for the moderator. You get the idea. This is the part of the process
where you start executing your plan.
Step 4. Collect Your Data
This is the meat and potatoes of your project; the time when you are administering your survey,
running your focus groups, conducting your interviews, implementing your field test, etc. The
answers, choices, and observations are all being collected and recorded, usually in spreadsheet
form. Each nugget of information is precious and will be part of the masterful conclusions you
will soon draw.
Ch Muhammad Irfan +92-345-4426176 facebook.com/chmuhammedirfan skype id:
ch.irfan786
Step 5. Analyze Your Data
Step 4 (data collection) has drawn to a close and you have heaps of raw data sitting in your lap. If
it’s on scraps of paper, you’ll probably need to get it in spreadsheet form for further analysis. If
it’s already in spreadsheet form, it’s time to make sure you’ve got it structured properly. Once
that’s all done, the fun begins.
Step 6. Visualize Your Data and Communicate Results
You’ve spent hours poring through your raw data, building useful summary tables, charts and
graphs. Now is the time to compile the most meaningful take-away into a digestible report or
presentation.
Topic:
Identify and define the various Legal Forms of Business which one you prefer
the best why:
Here are a range of legal structures associated with different forms of business:
Sole Proprietorship / Sole Traders:
Sole trader businesses are the easiest to set up because there is no complex paperwork. The
business and the owner are the same person in law. The sole trader does not have limited liability
which means that they are responsible for all the debts of the business. The sole trader has to
produce an annual accounting return for the Inland Revenue.
Partnerships:
Partnerships are set up by a Deed of Partnership which is a document made out by the partners
and witnessed by a solicitor. This Deed sets out the legal relationship between partners e.g. how
profits will be shared out, responsibilities of partners etc. In traditional partnerships the partners
had unlimited liability i.e. they were jointly responsible for the debts of their partnership. In 2001
this has been altered so that some large partnerships e.g. accountancy firms can have limited
liability.
Companies:
Companies are separate in law from the individual owners (shareholders) of the business. This
means that should the business run up debts the shareholders are only liable for these debts up
to the sum they have contributed to the company. A number of Companies Acts have been
passed setting out ways in which companies should conduct their affairs.
Which one Is Prefer:
For many small businesses, the best initial choice is either a sole proprietorship or, if more than
one owner is involved, a partnership. Either of these structures makes good sense in a business
where personal liability isn't a big worry -- for example, a small service business in which you are
unlikely to be sued and for which you won't be borrowing much money. Sole proprietorships and
partnerships are relatively simple and inexpensive to establish and maintain.
Topic:
Ch Muhammad Irfan +92-345-4426176 facebook.com/chmuhammedirfan skype id:
ch.irfan786
How would you build a management team and culture for your business
organization?
Building a strong culture within a team is at the core of business success. You want a culture that
recognizes and embraces shared values, attitudes, standards, and beliefs that characterize the
goals of the organization. And it’s a good idea to make sure it suits the best people who work at
the company while making a positive impression on customers and anyone else associated with
the business.
Following are some things I believe to be the cornerstones of a solid business culture.
Transparency:
At my company, we go over all the key metrics of the business with the entire company. The goal
is for all employees to feel they know the thinking, responsibilities, and strategy at various levels
of the company and can share ideas and feedback no matter who they are.
Time to disconnect.
We all need to hit the reset button once in a while -- people can't come in early and leave late
every single day without getting burnt out at some level. While you want employees to have a
work-hard founder’s mentality, you need to recognize the work-life integration that exists and
how significant it is to make sure you have personally fulfilled, clear-thinking people. It’s
important to understand that sometimes life will get in the way of business and everyone should
be allowed to take care of pressing personal matters.
Empowerment and a sense of freedom.
You empower people by not micromanaging, erring on the side of giving people general
guidelines rather than explicit, detailed directions. Informed employees are more involved and
empowered in a company. And the more freedom people have to take on tasks, manage them,
find solutions, and execute them, the more they feel connected to and woven into the company’s
culture.
 Physical space.
If you haven’t watched Susan Cane’s TED Talk on introverts, I highly recommend it. She opened
my mind to the idea that American businesses are built for extroverts, down to the floor plans of
our office spaces. Although open spaces are great for some, other people need to be able to close
the door to be at their most productive. It’s important to consider the comfort level of your
employees before you decide how to lay out space or what type of office space to lease.
Talking to customers and employees.
At different points in a company’s maturation process, you're almost guaranteed to go through
weeks or even months where you feel lost. When you haven’t nailed a product market fit or
you’re having challenges relating to your product or corporate vision, the natural tendency is to
turn your attention to where you or your team went wrong. Another way to try to solve these
problems is by talking to your customers.
Your organizational design.
Simply put, organizational design is the processes, structure, and hierarchy you put into place that
allow you to put your culture into practice. It’s “how you do things.” This will include your
Ch Muhammad Irfan +92-345-4426176 facebook.com/chmuhammedirfan skype id:
ch.irfan786
communication, company policies, team building, performance indicators, performance
evaluations, division of responsibilities, and even how you schedule, and run, meetings.
Topic:
Discuss the advantages and disadvantages:
An S corporation protects the personal assets of its shareholders. ... In a sole proprietorship or
general partnership, owners and the business are legally considered the same—leaving personal
assets vulnerable. Pass-through taxation. An S corporation does not pay federal taxes at
the corporate level.
Following are the advantages and disadvantages of S corporation:
 Eliminating double taxation:
In an S corporation, profits and losses are passed through to shareholders, and taxes are only
paid once. Check with your state to see how it handles S Corporations. Some states do not
recognize S Corporations and will tax such businesses as a regular C Corporation. Some states
charge S Corporations a state tax, although the corporation will not have to pay federal tax.
 Protection from liability:
As the owner of an S Corporation, your personal assets are separate from the business’s assets
and are therefore protected in case any judgments occur against the business.
 More room for investors:
S Corporations can have up to 100 shareholders.
 Easier accounting rules:
S Corporations without any inventory can use the cash method of accounting, which is much
simpler than the accrual method. Check with your accountant about which option makes sense
for your business.
Here are some disadvantages of forming an S Corporation:
 Rules and fees:
Like a C Corporation, S Corporations are required to file a number of official state and federal
documents, including Articles of Incorporation and corporate minutes. They must also hold
regular shareholder meetings and pay the required government fees.
 Shareholder restrictions:
Realize that if an S Corporation has shareholders, the shareholders will be taxed for any income
the company has, even if they did not receive any portion of that income. (In a C Corporation,
shareholders are taxed only if they receive dividends.) In addition, S Corporations are only
allowed to issue one class of stock, which may discourage some investors.
 Salary requirements:
The Internal Revenue Service requires all officers and owners of an S Corporation to make a
salary, even if the company is not yet making a profit. This could be problematic for new
businesses struggling to make payroll. A “reasonable salary” is what a person with the
appropriate skills needed for the position would be paid on the free market.
Topic:
Ch Muhammad Irfan +92-345-4426176 facebook.com/chmuhammedirfan skype id:
ch.irfan786
Briefly explain growth strategies based upon knowledge of product and or
market:
Growing a company takes a lot more than good intentions. You need a plan. With dozens of
strategies to choose from, here are four that could put your company on the fast track to growth.
Not every growth strategy is appropriate for every small business. The key to finding the right
growth strategy is properly matching it to your company and its specific marketplace. Since the
wrong strategy can devastate your business, it's important to determine whether you are selling
new or emerging products in a new or existing market.
Diversification (New Products/New Market)
Diversification is a high-risk growth strategy, largely because both the products and the market
are unproven territory for the entrepreneur. Though trailblazing emerging products and markets
can be exhilarating, it can also be terrifying given the fact that neither you nor anyone else can
rely on prior experience for reassurance. But if innovation is one of your company's defining
characteristics, a diversification strategy will eventually become second nature. To achieve
growth, you will need to be realistic about the risks you face and crystal clear about what you
hope to achieve.
Market Development (Existing Products/New Market)
A more common scenario is one in which a small business owner attempts to develop a new
market for their existing products and services. The new market can be geographical (e.g. foreign
export) or an untapped segment of a domestic market. It's even possible to develop a new market
for existing products by adjusting the product's packaging or expanding the product's distribution
channels. In any event, a market development growth strategy requires a working knowledge of
existing markets and the ability to gaps in the marketplace that can be exploited to your
advantage. If your marketing skills are not up to the task, you will need the assistance of a skilled
marketing professional to achieve growth in your new market.
Product Development (New Products/Existing Market)
A growth strategy based on product development is the mirror image of a market development
strategy. Instead of pioneering a new market with existing products, you attempt to roll out a new
product(s) in a market with which you are already familiar. Many small business owners are more
comfortable working in this kind of scenario because they already possess an awareness of
prevailing market conditions. However, a product development strategy can be just as challenging
as a market development strategy because it often requires the business to develop new abilities
and continuously adapt the products until they achieve marketplace success.
Market Penetration (Existing Products/Existing Markets)
Businesses that find themselves in a situation that involves neither new markets nor new
products are forced to grow through a market penetration strategy, a strategy that is designed to
give the business a greater percentage of market share. This type of strategy usually seeks to gain
a competitive edge through pricing, marketing, or other initiatives. Additionally, market
penetration can be achieved by increasing customer usage through loyalty programs and
incentives targeting your existing customer base.
Topic:
Ch Muhammad Irfan +92-345-4426176 facebook.com/chmuhammedirfan skype id:
ch.irfan786
Identify and define the major parts of a financial plan for a business:
Effective financial planning involves far more than balancing your checkbook on a monthly basis.
The ultimate goal of comprehensive financial planning is to gain financial clarity in all aspects of
your life.
How do you achieve financial clarity? By creating a ‘life plan’ that integrates your visions, values,
and motivations – all unique to you and your lifestyle – with your financial goals, realities, and
expectations.
1 Cash flow management
When was the last time you completed a cash flow summary or a statement of financial position?
To truly understand your current assets, liabilities, and net worth, it is important to identify –
concretely in writing – the status of your personal and professional income and expense balance
sheet. Other aspects of cash flow management include a debt elimination plan, if needed, as well
as a comprehensive savings plan.
2 Investment management
An in-depth, and again, regular review of your investments is another critical component of
financial planning. How your portfolio is designed, the strategies you have selected, your
tolerance for market risk and its effect on your portfolio returns – these should all be evaluated to
ensure that they are aligned with your investment goals, your time horizon, and your expected
lifestyle both today and in retirement. There are priorities and tradeoffs to be considered and
investment education from a trusted professional can be of value here.
3 Tax planning
In order to maximize and preserve your investment returns, an eye toward tax management is
crucial. There are a number of tax-reduction strategies, methods for generating tax-efficient
income, and also wealth transfer considerations – no matter your age – to understand and
implement in a proactive manner. For example, corporate bonds can benefit you more when held
in a tax-deferred account, while municipal bonds can better serve you in a taxable account.
4 Tax-deferred investment environments
Save early and save often. Take full advantage of any employer-sponsored retirement plan that is
offered to you. Whether as an employee, employer, or both, there are a number of investment
plan options that can help to augment your retirement savings. Talk to your financial advisor to
learn more about profit sharing plans, 401(k)s, IRAs, defined benefit plans, defined contribution
plans, as well as combination plan that could best serve your needs.
5 Insurance assessment
An important, and often overlooked, component of financial planning is an evaluation of the kinds
of insurance you need to protect yourself, your assets, and your loved ones. Insurance types can
include life, disability, umbrella, business overhead, and long-term care to name a few. Depending
on your stage in life, your insurance needs (aka risk management needs) will change and evolve.
6 Business succession planning
If you are a business owner, a significant portion of your financial plan will involve not only the
daily operations of your business, but also the eventual transfer or sale of your business to the
Ch Muhammad Irfan +92-345-4426176 facebook.com/chmuhammedirfan skype id:
ch.irfan786
next generation or a third party. An effective and tax-efficient transition requires foresight and
planning in order to maximize your benefits.
7 Revisit your written financial plan regularly
Most important to the success of your financial plan? Regular monitoring, personal accountability,
and course correction, as necessary, based on the unexpected curve balls that life will inevitably
throw your way. At the very least, an annual review with a financial planning professional will
keep you well positioned for, and informed regarding, any significant changes in your life
circumstances, objectives, or needs.
Topic:
Write a short note of the following:
Performa Cash Flow Statement:
The cash flow statement is an important tool, as it explains the changes in cash and gives the
information related to the business operating, investing and financing activities in a way to bring
advantage to short term analysis and cash planning of the business.
The basic objective of cash flow statement is to provide the information to the management
about the cash receipts and the cash payments of an organization being used for efficient cash
management. Since one of the important functions of a financial manager is cash management
and to ensure if adequate cash is available to meet the liabilities, it is owing to the cash flow
statement that the related information is derived. In this way, cash flow statement is useful to
plan financial operations in an efficient manner.
Cash flows are inflows and outflows of cash and cash equivalents. The cash activities are classified
into three main categories of cash inflows and cash outflows. The tree categories are: -
1. Operating activities
2. Investing Activities
3. Financing Activities
Break Even Analysis:
A breakeven analysis is used to determine how much sales volume your business needs to start
making a profit. The breakeven analysis is especially useful when you're developing a pricing
strategy, either as part of a marketing plan or a business plan. To conduct a breakeven analysis,
use this formula:
Fixed Costs divided by (Revenue per unit - Variable costs per unit)
Fixed costs are costs that must be paid whether or not any units are produced. These costs are
"fixed" over a specified period of time or range of production.
Variable costs are costs that vary directly with the number of products produced. For instance, the
cost of the materials needed and the labor used to produce units isn't always the same.
Topic:
Briefly explain intellectual property and trade mark:
Trademarks:
Trade marks protect any symbol that indicates the source or origin of the goods or services to
which it is affixed. While a trademark can be extremely valuable to its owner, the ultimate
Ch Muhammad Irfan +92-345-4426176 facebook.com/chmuhammedirfan skype id:
ch.irfan786
purpose of a trademark is to protect consumers – that is, the function of a trademark is to inform
the consumer where the goods or services originate. The consumer, knowing the origin of the
goods, can make purchasing decisions based on prior knowledge, reputation or marketing.
Intellectual property:
Intellectual property is a broad categorical description for the set of intangibles owned and legally
protected by a company from outside use or implementation without consent. Intellectual
property can consist of patents, trade secrets, copyrights and trademarks, or simply ideas.
The concept of intellectual property relates to the fact that certain products of human intellect
should be afforded the same protective rights that apply to physical property. Most developed
economies have legal measures in place to protect both forms of property.
Topic:
Identify and define the major source of capital for your business:
Often the hardest part of starting a business is raising the money to get going. The entrepreneur
might have a great idea and clear idea of how to turn it into a successful business. However, if
sufficient finance can't be raised, it is unlikely that the business will get off the ground.
Your savings
Most entrepreneurs start their companies by investing their own savings. This source of financing
can be ideal – if you can afford it. It puts you in full control of how much you are going to get.
Furthermore, you never have to justify yourself to investors. This last point is an important
benefit. You have the freedom to operate as you see best. There is a trade-off, though: this
freedom usually comes at the expense of having little money.
Your credit
Many entrepreneurs also use their credit – either through credit cards or a home equity loan – to
start their small businesses. Be very careful about using these sources, as you could ruin your
personal credit, risk your home, or both. If possible, avoid using a home equity line of credit as the
risk is too high.
Family members and friends
One common way to finance a business is to ask friends and family members for an investment.
Because this method is probably one of the riskiest ways to finance a business, we recommend
against it. The problem is that if things go wrong, your friend/family relationship is affected. And
in any startup business, you are guaranteed that things will go wrong at one point or another.
Customers and suppliers
Two great funding sources for small businesses are actually your customers and your suppliers.
Customers can finance your business by prepaying for orders – or by giving you an advance.
Obviously, you use these funds to handle their transaction. This funding is not easy to get, but
customers may be willing to prepay if your service is unique enough or if they really need your
products.
Small Business Administration
Ch Muhammad Irfan +92-345-4426176 facebook.com/chmuhammedirfan skype id:
ch.irfan786
One great source of small business funding is the Small Business Administration (SBA). The SBA
has a special program that offers Microloans to small business owners. Microloans can
reach $50,000 (varies by state) and are much easier to get than conventional financing. They also
come bundled with business training which can be very useful for first-time entrepreneurs. The
SBA also works with banks to provide (a) loans. These loans are larger that Microloans and have
stricter underwriting criteria.
Peer-to-peer lenders
Peer-to-peer (P2P) lenders offer a platform that allows people to seek financing from other
individuals. The P2P platform acts as a matching service and provides basic due diligence. Like
Microloans, most P2P loans are small. They usually cap at $25,000 to $35,000.
 Leasing
Small businesses in certain industries need equipment to get started. One way to finance the
equipment is to lease it through a finance company. Most equipment leases are structured so
that the finance company buys the equipment and rents it to you for a monthly payment. Once
the term of the lease concludes, you can buy the equipment from the finance company for a fairly
modest cost.
Topic:
What Is the Difference Between a Marketing & Business Plan?
The business plan and the marketing plan are both essential aspects of successful businesses.
These plans not only help businesses organize their operations, but help benchmark their success
or failure. Since business industries change with customer demand and the economic
environment, the business plan and marketing plan must evolve throughout the lifespan of the
business.
Business Plan:
A written document describing the nature of the business, the sales and marketing strategy, and
the financial background, and containing a projected profit and loss statement.
A business plan is also a road map that provides directions so a business can plan its future and
helps it avoid bumps in the road. The time you spend making your business plan thorough and
accurate, and keeping it up-to-date, is an investment that pays big dividends in the long term.
Your business plan should conform to generally accepted guidelines regarding form and content.
Each section should include specific elements and address relevant questions that the people who
read your plan will most likely ask.
Marketing Plan:
A marketing plan is a business document written for the purpose of describing the current market
position of a business and its marketing strategy for the period covered by the marketing plan.
Marketing plans usually cover a period of one to five years. The purpose of a marketing plan is to
clearly show what steps or actions will be taken to achieve the plan goals. For example, a
marketing plan might have a strategy to increase the organization's market share by ten percent.
The plan would then outline the goals that need to be achieved in order to reach a ten percent
increase in market share.
Ch Muhammad Irfan +92-345-4426176 facebook.com/chmuhammedirfan skype id:
ch.irfan786
Topic
What are the basic principles of Time management?
Here’s an overview of the seven principles of time management as outlined by Mark in the book:
Have a clear vision
 Ask yourself "What am I actually trying to achieve?"
 The clearer you are about your vision, the more likely you are to achieve it.
 It’s as much about what you’re not going to do as it is about what you are –- you are
establishing limits. For example, "I will process my emails for no more than 20 minutes."
Do one thing at a time
 Successful people don’t take on too much. They concentrate on doing and finishing one
thing at a time as far as possible.
 Little and often -- the human mind works best when we apply this principle.
Define your limits
 The best way to be creative is not to try to think without limits, but to carefully define what
those limits should be. Limitations actually encourage creativity.
 If you feel you can’t get going or you're getting nowhere, it’s probably due to poorly
defined limits. For instance, if you have a limited amount of time you will be able to
concentrate your efforts better than if you have unlimited time.
Use Closed lists
 Any list that has a line drawn at the end of it can’t be added to. This enables you to deal
with all the items on it without the distraction of new work being added.
 Once defined, it can only stay the same or get smaller.
 It doesn’t matter which order you do things, provided you are going to clear the whole list.
 If you have a backlog of work to deal with, use the closed list principle.
Reduced random factors
These are things that stop us from completing our planned actions and can never be eliminated.
Your day starts to run you rather than you running it. We tend to react to random elements; that
is, we use the reactive part of our brain to react to who or whatever makes the most ‘noise’. We
tend to prioritize by noise.
Topic:
How would you overcome the pressure on Existing Financial Resources?
Overcoming Pressures on Existing Financial Resources
 To overcome pressures, the entrepreneur could acquire new resources.
 The acquisition of new resources is expensive, whether in terms of the equity sold or the
interest payments from debt.
 The need or the magnitude of the new resources required can be reduced through better
management of existing resources.
Managing Cash Flow
Ch Muhammad Irfan +92-345-4426176 facebook.com/chmuhammedirfan skype id:
ch.irfan786
 The entrepreneur should have an up-to-date assessment of the cash position.
 A daily cash sheet would provide an effective indication of any daily shortfall and of
problems or errors that might have occurred.
 Compare budgeted or expected cash flows with actual cash flows
Managing Inventory
 Perpetual inventory systems can be structured using computers or a manual system.
 To check the inventory balance, it may be necessary to physically count inventory
periodically.
 Link the needs of a retailer with the wholesaler and producer allowing for a fast order
entry and response.
 Transport mode selection can also be important.
Managing Fixed Assets
 Generally, involve long-term commitments and large investments for the new venture.
 Equipment will require servicing and insurance and affect utility costs; will also depreciate
over time.
 Leasing can be an alternative to buying depending on:
 Terms of the lease.
 Type of asset.
 Usage demand.
 Lease payments can be used as a tax deduction.
Taxes
 Withhold federal and state taxes for employees.
 Pay a number of taxes (state and federal unemployment taxes and business taxes).
 Allocate taxes as part of any budget.
 File end-of-year returns of the business.
 Consider use of a tax accountant.
Topic:
Enterprenurial management is distinct from Traditional management in
terms of eight dimensions. Identify and explain
Entrepreneur:
He is a person who establishes business unit and utilizes the other factors of production like land,
labor and capital.
Manager:
A manager is the person responsible for planning & directing the work of a group of individuals,
monitoring their work, and taking corrective action when necessary.
PRIMARY MOTIVES:
CHARACTERISTICS ENTREPRENEUR TRADITIONAL MANAGER
Ch Muhammad Irfan +92-345-4426176 facebook.com/chmuhammedirfan skype id:
ch.irfan786
PRIMARY
MOTIVES
Wants freedom, goal oriented,
self-reliant, and self-motivated
Wants promotion and other
traditional corporate
rewards power motivated,
TIME
ORIENTATION
Uses and goals of 5 to 10-year
growth of the business as
guides; takes action how to
next step along the way
Responds to quotes and
budgets ; to weekly,
monthly, quarterly, and
annual planning horizons;
and to the next promotion
or transfer,
TENDENCY TO
ACTION
Gets hand dirty; may upset
employee by suddenly doing
their work,
Delegates action;
supervising and reporting
takes most energy,
SKILLS Knows business intimately;
more business acumen then
managerial or political skills;
often technically trained if in
technical business; may have
had profit and loss
responsibility in the company,
Professional management ;
often business school
trained; uses abstract
analytical tools, people-
management and political
skills,
ATTITUDE
TOWARDS
COURAGE AND
DESTINY
Self-confident, optimistic and
courageous,
Self-confident and
courageous; many are
cynical about the system
but optimistic about their
ability to outwit it.
FOCUS OF
ATTENTION
Primarily on technology and
marketplace,
Primarily on events inside
corporation,
ATTITUDE
TOWARDS RISK
Likes moderate risk; invests
heavily but expects to succeed,
Cautions,
USE OF MARKET
RESEARCH
Creates needs; creates
products that often cannot be
tested with market research;
potential customers do not yet
understand them; talks to
Has market studies done to
discover needs and guide
product conceptualization,
Ch Muhammad Irfan +92-345-4426176 facebook.com/chmuhammedirfan skype id:
ch.irfan786
customers and forms own
opinion,
Topic:
As an entrepreneur how do you minimize Patent risk in the business?
 Seek a patent attorney who has expertise in your product line.
 The entrepreneur should consider a design patent to protect the product design or product
look.
 Before making an external disclosure of an invention at a conference or to the media, or
before setting up a beta site, the entrepreneur should seek legal counsel since this external
disclosure may negate a subsequent patent application.
 Evaluate competitor patents to gain insight into what they may be developing.
 If you think your product infringes on the patent of another firm, seek legal counsel.
 Verify that all employment contracts with individuals who may contribute new products
 have clauses assigning those inventions or new products to the venture.
 Be sure to properly mark all products granted a patent. Not having products marked
 could result in loss or damages in a patent suit.
 Consider licensing your patents. This can enhance the investment in a patent by creating
 new market opportunities and can increase long-term revenue.

Contenu connexe

Similaire à Entrepreneurship

Qualities of an entrepreneur
Qualities of an entrepreneurQualities of an entrepreneur
Qualities of an entrepreneurL.Prakash Kannan
 
WEEK-2 - INTRO TO ENTREPRENEURSHIP .pptx
WEEK-2 - INTRO TO ENTREPRENEURSHIP .pptxWEEK-2 - INTRO TO ENTREPRENEURSHIP .pptx
WEEK-2 - INTRO TO ENTREPRENEURSHIP .pptxPatrisiaPadilla
 
Chapter 04-Characteristics of Successful Entrepreneurs
Chapter 04-Characteristics of Successful EntrepreneursChapter 04-Characteristics of Successful Entrepreneurs
Chapter 04-Characteristics of Successful EntrepreneursAbdinasirHassan4
 
Entrepreneurship development
Entrepreneurship developmentEntrepreneurship development
Entrepreneurship developmentSiraj Khan
 
Entrepreneurship developmen book in mku syllabus hariharan23900
Entrepreneurship developmen book in mku syllabus   hariharan23900Entrepreneurship developmen book in mku syllabus   hariharan23900
Entrepreneurship developmen book in mku syllabus hariharan23900hariharan n
 
1st Enterpreneurship, M.Com
1st Enterpreneurship, M.Com1st Enterpreneurship, M.Com
1st Enterpreneurship, M.ComDanish Saqi
 
entrepreneurialskills.pptx
entrepreneurialskills.pptxentrepreneurialskills.pptx
entrepreneurialskills.pptxbrinda61
 
Introduction to ENTREPRENEURSHIP
Introduction to ENTREPRENEURSHIPIntroduction to ENTREPRENEURSHIP
Introduction to ENTREPRENEURSHIPAbdul Basit
 
5 Suggestions to Building a More Successful Business
5 Suggestions to Building a More Successful Business5 Suggestions to Building a More Successful Business
5 Suggestions to Building a More Successful BusinessJim Jorgensen
 
Essential Characteristics of Entrepreneurs
Essential Characteristics of EntrepreneursEssential Characteristics of Entrepreneurs
Essential Characteristics of EntrepreneursHimansu S Mahapatra
 
Module 2 creative me
Module 2  creative meModule 2  creative me
Module 2 creative meyeaproject
 
Becoming an-entrepreneur
Becoming an-entrepreneurBecoming an-entrepreneur
Becoming an-entrepreneurDavid Kiama
 
Introduction to entrepreneurship
Introduction to entrepreneurshipIntroduction to entrepreneurship
Introduction to entrepreneurshipAbby Gale
 
ENTREPRENEURSHIP DEVELOPMENT LAB SM54
ENTREPRENEURSHIP DEVELOPMENT LAB SM54ENTREPRENEURSHIP DEVELOPMENT LAB SM54
ENTREPRENEURSHIP DEVELOPMENT LAB SM54Subhash Mahla
 

Similaire à Entrepreneurship (20)

Qualities of an entrepreneur
Qualities of an entrepreneurQualities of an entrepreneur
Qualities of an entrepreneur
 
WEEK-2 - INTRO TO ENTREPRENEURSHIP .pptx
WEEK-2 - INTRO TO ENTREPRENEURSHIP .pptxWEEK-2 - INTRO TO ENTREPRENEURSHIP .pptx
WEEK-2 - INTRO TO ENTREPRENEURSHIP .pptx
 
Chapter 04-Characteristics of Successful Entrepreneurs
Chapter 04-Characteristics of Successful EntrepreneursChapter 04-Characteristics of Successful Entrepreneurs
Chapter 04-Characteristics of Successful Entrepreneurs
 
Entrepreneurship development
Entrepreneurship developmentEntrepreneurship development
Entrepreneurship development
 
Entrepreneurship developmen book in mku syllabus hariharan23900
Entrepreneurship developmen book in mku syllabus   hariharan23900Entrepreneurship developmen book in mku syllabus   hariharan23900
Entrepreneurship developmen book in mku syllabus hariharan23900
 
1st Enterpreneurship, M.Com
1st Enterpreneurship, M.Com1st Enterpreneurship, M.Com
1st Enterpreneurship, M.Com
 
Entrpreneurship
EntrpreneurshipEntrpreneurship
Entrpreneurship
 
module 1.pptx
module 1.pptxmodule 1.pptx
module 1.pptx
 
entrepreneurialskills.pptx
entrepreneurialskills.pptxentrepreneurialskills.pptx
entrepreneurialskills.pptx
 
Introduction to ENTREPRENEURSHIP
Introduction to ENTREPRENEURSHIPIntroduction to ENTREPRENEURSHIP
Introduction to ENTREPRENEURSHIP
 
Entrepreneurship development
Entrepreneurship developmentEntrepreneurship development
Entrepreneurship development
 
5 Suggestions to Building a More Successful Business
5 Suggestions to Building a More Successful Business5 Suggestions to Building a More Successful Business
5 Suggestions to Building a More Successful Business
 
Entrepreneurial skills
Entrepreneurial skillsEntrepreneurial skills
Entrepreneurial skills
 
Essential Characteristics of Entrepreneurs
Essential Characteristics of EntrepreneursEssential Characteristics of Entrepreneurs
Essential Characteristics of Entrepreneurs
 
Module 2 creative me
Module 2  creative meModule 2  creative me
Module 2 creative me
 
Module 2 Creative Me
Module 2  Creative MeModule 2  Creative Me
Module 2 Creative Me
 
Becoming an-entrepreneur
Becoming an-entrepreneurBecoming an-entrepreneur
Becoming an-entrepreneur
 
Introduction to entrepreneurship
Introduction to entrepreneurshipIntroduction to entrepreneurship
Introduction to entrepreneurship
 
Lecture 3-4.pptx
Lecture 3-4.pptxLecture 3-4.pptx
Lecture 3-4.pptx
 
ENTREPRENEURSHIP DEVELOPMENT LAB SM54
ENTREPRENEURSHIP DEVELOPMENT LAB SM54ENTREPRENEURSHIP DEVELOPMENT LAB SM54
ENTREPRENEURSHIP DEVELOPMENT LAB SM54
 

Plus de Ch Irfan

Management Information System
Management Information SystemManagement Information System
Management Information SystemCh Irfan
 
Knowledge management
Knowledge managementKnowledge management
Knowledge managementCh Irfan
 
Strategic management
Strategic managementStrategic management
Strategic managementCh Irfan
 
Human Resource Management II
Human Resource Management IIHuman Resource Management II
Human Resource Management IICh Irfan
 
Human Resource management I
Human Resource management IHuman Resource management I
Human Resource management ICh Irfan
 
International management
International management International management
International management Ch Irfan
 
Principles of marketing
Principles of marketingPrinciples of marketing
Principles of marketingCh Irfan
 
Organizational behaviour part 02
Organizational behaviour part 02 Organizational behaviour part 02
Organizational behaviour part 02 Ch Irfan
 
Orgnaizational behaviour part 01
Orgnaizational behaviour part 01Orgnaizational behaviour part 01
Orgnaizational behaviour part 01Ch Irfan
 
Financial institution
Financial institutionFinancial institution
Financial institutionCh Irfan
 
Project management
Project managementProject management
Project managementCh Irfan
 
Research method
Research methodResearch method
Research methodCh Irfan
 
Principles of management
Principles of managementPrinciples of management
Principles of managementCh Irfan
 
Orgnaizational communication
Orgnaizational communicationOrgnaizational communication
Orgnaizational communicationCh Irfan
 

Plus de Ch Irfan (14)

Management Information System
Management Information SystemManagement Information System
Management Information System
 
Knowledge management
Knowledge managementKnowledge management
Knowledge management
 
Strategic management
Strategic managementStrategic management
Strategic management
 
Human Resource Management II
Human Resource Management IIHuman Resource Management II
Human Resource Management II
 
Human Resource management I
Human Resource management IHuman Resource management I
Human Resource management I
 
International management
International management International management
International management
 
Principles of marketing
Principles of marketingPrinciples of marketing
Principles of marketing
 
Organizational behaviour part 02
Organizational behaviour part 02 Organizational behaviour part 02
Organizational behaviour part 02
 
Orgnaizational behaviour part 01
Orgnaizational behaviour part 01Orgnaizational behaviour part 01
Orgnaizational behaviour part 01
 
Financial institution
Financial institutionFinancial institution
Financial institution
 
Project management
Project managementProject management
Project management
 
Research method
Research methodResearch method
Research method
 
Principles of management
Principles of managementPrinciples of management
Principles of management
 
Orgnaizational communication
Orgnaizational communicationOrgnaizational communication
Orgnaizational communication
 

Dernier

Organizational Structure Running A Successful Business
Organizational Structure Running A Successful BusinessOrganizational Structure Running A Successful Business
Organizational Structure Running A Successful BusinessSeta Wicaksana
 
Kenya Coconut Production Presentation by Dr. Lalith Perera
Kenya Coconut Production Presentation by Dr. Lalith PereraKenya Coconut Production Presentation by Dr. Lalith Perera
Kenya Coconut Production Presentation by Dr. Lalith Pereraictsugar
 
The CMO Survey - Highlights and Insights Report - Spring 2024
The CMO Survey - Highlights and Insights Report - Spring 2024The CMO Survey - Highlights and Insights Report - Spring 2024
The CMO Survey - Highlights and Insights Report - Spring 2024christinemoorman
 
Digital Transformation in the PLM domain - distrib.pdf
Digital Transformation in the PLM domain - distrib.pdfDigital Transformation in the PLM domain - distrib.pdf
Digital Transformation in the PLM domain - distrib.pdfJos Voskuil
 
Intro to BCG's Carbon Emissions Benchmark_vF.pdf
Intro to BCG's Carbon Emissions Benchmark_vF.pdfIntro to BCG's Carbon Emissions Benchmark_vF.pdf
Intro to BCG's Carbon Emissions Benchmark_vF.pdfpollardmorgan
 
Independent Call Girls Andheri Nightlaila 9967584737
Independent Call Girls Andheri Nightlaila 9967584737Independent Call Girls Andheri Nightlaila 9967584737
Independent Call Girls Andheri Nightlaila 9967584737Riya Pathan
 
Future Of Sample Report 2024 | Redacted Version
Future Of Sample Report 2024 | Redacted VersionFuture Of Sample Report 2024 | Redacted Version
Future Of Sample Report 2024 | Redacted VersionMintel Group
 
Call Us 📲8800102216📞 Call Girls In DLF City Gurgaon
Call Us 📲8800102216📞 Call Girls In DLF City GurgaonCall Us 📲8800102216📞 Call Girls In DLF City Gurgaon
Call Us 📲8800102216📞 Call Girls In DLF City Gurgaoncallgirls2057
 
8447779800, Low rate Call girls in Saket Delhi NCR
8447779800, Low rate Call girls in Saket Delhi NCR8447779800, Low rate Call girls in Saket Delhi NCR
8447779800, Low rate Call girls in Saket Delhi NCRashishs7044
 
Investment in The Coconut Industry by Nancy Cheruiyot
Investment in The Coconut Industry by Nancy CheruiyotInvestment in The Coconut Industry by Nancy Cheruiyot
Investment in The Coconut Industry by Nancy Cheruiyotictsugar
 
Call Girls Miyapur 7001305949 all area service COD available Any Time
Call Girls Miyapur 7001305949 all area service COD available Any TimeCall Girls Miyapur 7001305949 all area service COD available Any Time
Call Girls Miyapur 7001305949 all area service COD available Any Timedelhimodelshub1
 
FULL ENJOY Call girls in Paharganj Delhi | 8377087607
FULL ENJOY Call girls in Paharganj Delhi | 8377087607FULL ENJOY Call girls in Paharganj Delhi | 8377087607
FULL ENJOY Call girls in Paharganj Delhi | 8377087607dollysharma2066
 
8447779800, Low rate Call girls in Kotla Mubarakpur Delhi NCR
8447779800, Low rate Call girls in Kotla Mubarakpur Delhi NCR8447779800, Low rate Call girls in Kotla Mubarakpur Delhi NCR
8447779800, Low rate Call girls in Kotla Mubarakpur Delhi NCRashishs7044
 
Contemporary Economic Issues Facing the Filipino Entrepreneur (1).pptx
Contemporary Economic Issues Facing the Filipino Entrepreneur (1).pptxContemporary Economic Issues Facing the Filipino Entrepreneur (1).pptx
Contemporary Economic Issues Facing the Filipino Entrepreneur (1).pptxMarkAnthonyAurellano
 
Flow Your Strategy at Flight Levels Day 2024
Flow Your Strategy at Flight Levels Day 2024Flow Your Strategy at Flight Levels Day 2024
Flow Your Strategy at Flight Levels Day 2024Kirill Klimov
 
Buy gmail accounts.pdf Buy Old Gmail Accounts
Buy gmail accounts.pdf Buy Old Gmail AccountsBuy gmail accounts.pdf Buy Old Gmail Accounts
Buy gmail accounts.pdf Buy Old Gmail AccountsBuy Verified Accounts
 
8447779800, Low rate Call girls in Shivaji Enclave Delhi NCR
8447779800, Low rate Call girls in Shivaji Enclave Delhi NCR8447779800, Low rate Call girls in Shivaji Enclave Delhi NCR
8447779800, Low rate Call girls in Shivaji Enclave Delhi NCRashishs7044
 
Marketplace and Quality Assurance Presentation - Vincent Chirchir
Marketplace and Quality Assurance Presentation - Vincent ChirchirMarketplace and Quality Assurance Presentation - Vincent Chirchir
Marketplace and Quality Assurance Presentation - Vincent Chirchirictsugar
 
BEST Call Girls In Greater Noida ✨ 9773824855 ✨ Escorts Service In Delhi Ncr,
BEST Call Girls In Greater Noida ✨ 9773824855 ✨ Escorts Service In Delhi Ncr,BEST Call Girls In Greater Noida ✨ 9773824855 ✨ Escorts Service In Delhi Ncr,
BEST Call Girls In Greater Noida ✨ 9773824855 ✨ Escorts Service In Delhi Ncr,noida100girls
 

Dernier (20)

Organizational Structure Running A Successful Business
Organizational Structure Running A Successful BusinessOrganizational Structure Running A Successful Business
Organizational Structure Running A Successful Business
 
Kenya Coconut Production Presentation by Dr. Lalith Perera
Kenya Coconut Production Presentation by Dr. Lalith PereraKenya Coconut Production Presentation by Dr. Lalith Perera
Kenya Coconut Production Presentation by Dr. Lalith Perera
 
The CMO Survey - Highlights and Insights Report - Spring 2024
The CMO Survey - Highlights and Insights Report - Spring 2024The CMO Survey - Highlights and Insights Report - Spring 2024
The CMO Survey - Highlights and Insights Report - Spring 2024
 
Digital Transformation in the PLM domain - distrib.pdf
Digital Transformation in the PLM domain - distrib.pdfDigital Transformation in the PLM domain - distrib.pdf
Digital Transformation in the PLM domain - distrib.pdf
 
Intro to BCG's Carbon Emissions Benchmark_vF.pdf
Intro to BCG's Carbon Emissions Benchmark_vF.pdfIntro to BCG's Carbon Emissions Benchmark_vF.pdf
Intro to BCG's Carbon Emissions Benchmark_vF.pdf
 
Japan IT Week 2024 Brochure by 47Billion (English)
Japan IT Week 2024 Brochure by 47Billion (English)Japan IT Week 2024 Brochure by 47Billion (English)
Japan IT Week 2024 Brochure by 47Billion (English)
 
Independent Call Girls Andheri Nightlaila 9967584737
Independent Call Girls Andheri Nightlaila 9967584737Independent Call Girls Andheri Nightlaila 9967584737
Independent Call Girls Andheri Nightlaila 9967584737
 
Future Of Sample Report 2024 | Redacted Version
Future Of Sample Report 2024 | Redacted VersionFuture Of Sample Report 2024 | Redacted Version
Future Of Sample Report 2024 | Redacted Version
 
Call Us 📲8800102216📞 Call Girls In DLF City Gurgaon
Call Us 📲8800102216📞 Call Girls In DLF City GurgaonCall Us 📲8800102216📞 Call Girls In DLF City Gurgaon
Call Us 📲8800102216📞 Call Girls In DLF City Gurgaon
 
8447779800, Low rate Call girls in Saket Delhi NCR
8447779800, Low rate Call girls in Saket Delhi NCR8447779800, Low rate Call girls in Saket Delhi NCR
8447779800, Low rate Call girls in Saket Delhi NCR
 
Investment in The Coconut Industry by Nancy Cheruiyot
Investment in The Coconut Industry by Nancy CheruiyotInvestment in The Coconut Industry by Nancy Cheruiyot
Investment in The Coconut Industry by Nancy Cheruiyot
 
Call Girls Miyapur 7001305949 all area service COD available Any Time
Call Girls Miyapur 7001305949 all area service COD available Any TimeCall Girls Miyapur 7001305949 all area service COD available Any Time
Call Girls Miyapur 7001305949 all area service COD available Any Time
 
FULL ENJOY Call girls in Paharganj Delhi | 8377087607
FULL ENJOY Call girls in Paharganj Delhi | 8377087607FULL ENJOY Call girls in Paharganj Delhi | 8377087607
FULL ENJOY Call girls in Paharganj Delhi | 8377087607
 
8447779800, Low rate Call girls in Kotla Mubarakpur Delhi NCR
8447779800, Low rate Call girls in Kotla Mubarakpur Delhi NCR8447779800, Low rate Call girls in Kotla Mubarakpur Delhi NCR
8447779800, Low rate Call girls in Kotla Mubarakpur Delhi NCR
 
Contemporary Economic Issues Facing the Filipino Entrepreneur (1).pptx
Contemporary Economic Issues Facing the Filipino Entrepreneur (1).pptxContemporary Economic Issues Facing the Filipino Entrepreneur (1).pptx
Contemporary Economic Issues Facing the Filipino Entrepreneur (1).pptx
 
Flow Your Strategy at Flight Levels Day 2024
Flow Your Strategy at Flight Levels Day 2024Flow Your Strategy at Flight Levels Day 2024
Flow Your Strategy at Flight Levels Day 2024
 
Buy gmail accounts.pdf Buy Old Gmail Accounts
Buy gmail accounts.pdf Buy Old Gmail AccountsBuy gmail accounts.pdf Buy Old Gmail Accounts
Buy gmail accounts.pdf Buy Old Gmail Accounts
 
8447779800, Low rate Call girls in Shivaji Enclave Delhi NCR
8447779800, Low rate Call girls in Shivaji Enclave Delhi NCR8447779800, Low rate Call girls in Shivaji Enclave Delhi NCR
8447779800, Low rate Call girls in Shivaji Enclave Delhi NCR
 
Marketplace and Quality Assurance Presentation - Vincent Chirchir
Marketplace and Quality Assurance Presentation - Vincent ChirchirMarketplace and Quality Assurance Presentation - Vincent Chirchir
Marketplace and Quality Assurance Presentation - Vincent Chirchir
 
BEST Call Girls In Greater Noida ✨ 9773824855 ✨ Escorts Service In Delhi Ncr,
BEST Call Girls In Greater Noida ✨ 9773824855 ✨ Escorts Service In Delhi Ncr,BEST Call Girls In Greater Noida ✨ 9773824855 ✨ Escorts Service In Delhi Ncr,
BEST Call Girls In Greater Noida ✨ 9773824855 ✨ Escorts Service In Delhi Ncr,
 

Entrepreneurship

  • 1. Ch Muhammad Irfan +92-345-4426176 facebook.com/chmuhammedirfan skype id: ch.irfan786 Topic: How do role model and support system influence entrepreneurs in their career? Role models -Individuals influencing an entrepreneur’s career choice and style  Can be parents, family members, or other entrepreneurs.  Successful entrepreneurs are viewed as catalysts by potential entrepreneurs.  Role models can serve in a supportive capacity as mentors by providing information, advice and guidance.  Entrepreneurs need to establish connections and networks in the venture formation process Moral-Support Network It is important for the entrepreneur to establish a moral support network of family and friends. Most entrepreneurs indicate that their spouses are their biggest supporters. Friends can provide advice that is more honest than that received from others, plus encouragement, understanding, and assistance. Relatives can also be sources of moral support, particularly if they are also entrepreneurs. Professional-Support Network The entrepreneur also needs advice and counsel, which can be obtained from members of a professional support network. Topic: List the characteristics that you believe are necessary for a successful entrepreneur: 1. Disciplined These individuals are focused on making their businesses work, and eliminate any hindrances or distractions to their goals. They have overarching strategies and outline the tactics to accomplish them. Successful entrepreneurs are disciplined enough to take steps every day toward the achievement of their objectives. 2. Confidence The entrepreneur does not ask questions about whether they can succeed or whether they are worthy of success. They are confident with the knowledge that they will make their businesses succeed. They exude that confidence in everything they do. 3. Open Minded Entrepreneurs realize that every event and situation is a business opportunity. Ideas are constantly being generated about workflows and efficiency, people skills and potential new businesses. They have the ability to look at everything around them and focus it toward their goals. 4. Self-Starter Entrepreneurs know that if something needs to be done, they should start it themselves. They set the parameters and make sure that projects follow that path. They are proactive, not waiting for someone to give them permission. 5. Competitive
  • 2. Ch Muhammad Irfan +92-345-4426176 facebook.com/chmuhammedirfan skype id: ch.irfan786 Many companies are formed because an entrepreneur knows that they can do a job better than another. They need to win at the sports they play and need to win at the businesses that they create. An entrepreneur will highlight their own company’s track record of success. 6. Creativity One facet of creativity is being able to make connections between seemingly unrelated events or situations. Entrepreneurs often come up with solutions which are the synthesis of other items. They will repurpose products to market them to new industries. 7. Determination Entrepreneurs are not thwarted by their defeats. They look at defeat as an opportunity for success. They are determined to make all of their endeavors succeed, so will try and try again until it does. Successful entrepreneurs do not believe that something cannot be done. 8. Strong people skills The entrepreneur has strong communication skills to sell the product and motivate employees. Most successful entrepreneurs know how to motivate their employees so the business grows overall. They are very good at highlighting the benefits of any situation and coaching others to their success. 9. Strong work ethic The successful entrepreneur will often be the first person to arrive at the office and the last one to leave. They will come in on their days off to make sure that an outcome meets their expectations. Their mind is constantly on their work, whether they are in or out of the workplace. 10. Passion Passion is the most important trait of the successful entrepreneur. They genuinely love their work. They are willing to put in those extra hours to make the business succeed because there is a joy their business gives which goes beyond the money. The successful entrepreneur will always be reading and researching ways to make the business better. Topic: Write a note on how entrepreneurs think:  Entrepreneurs think by actions: The entrepreneurial mindset desires action, so much so that they twist the age old saw to read "Ready, Fire, Aim." Action for the entrepreneur tests their hypotheses and so provides them with special market intelligence regardless of the outcome. Entrepreneurs think in possibilities: Entrepreneurial thinking is not constrained by the reality of the present. Entrepreneurs see the reality of possibilities that are not yet evident to others. As HBS professor Howard Stevenson explains: "Entrepreneurs pursue opportunities without regard to resources currently controlled." Entrepreneurs don't need all the resources in place before pursuing an opportunity ... just the important ones.  Entrepreneurs can think in ambiguity: For the vast majority of people, ambiguity means confusion, disorganization and debilitation. Not so for the entrepreneur who can still function effectively and move forward decisively in the world of conflicting views, uncertain outcomes, and unresolved issues that exists as they craft and hone their opportunity from the evolving mass of possibilities. The Lemonade entrepreneurial
  • 3. Ch Muhammad Irfan +92-345-4426176 facebook.com/chmuhammedirfan skype id: ch.irfan786 principle actually embraces surprises that arise from this uncertainty and will adjust goals on the fly rather than be held by previous ones.  Entrepreneurial thinking embraces risk: While many money makers implement risk avoidance strategies to create wealth, the entrepreneur knows that opportunities are more likely to exist for them by embracing and managing risk rather than simply avoiding it. It is in these risky areas that the established players avoid, that contains the gold nugget opportunities. Entrepreneurs have developed a whole host of risk management treatments to help them in this regard like lean startups, customer development process, beta launches, iterations and pivots, affordable loss experimentation, 'under the radar' launches, embedded entrepreneurship, and beachhead strategies. The Affordable Loss Principle drives entrepreneurial action based on the acceptable downside, more than it does on the attractiveness of the predicted upside.  Entrepreneurs think in patterns: Entrepreneurs are always looking for cause and effect patterns in the world around them. Patterns help the entrepreneur to not only understand the world they live in but it also helps them predict the future and to see opportunities that most people miss. These patterns may be the ability to see the logical extension of a change or the creative-disruptive force of something new or the cross pollination ability of an innovation into new fields. Entrepreneurs also know how to apply the concepts and patterns from existing worlds to explain innovations in worlds as yet unseen.  Entrepreneurs think incessantly: There is no off button in the entrepreneur's mind. They are incessantly curious, opportunistic, and optimistic. They train their minds to problem-solve every issue that crosses their path. Every person they meet is a potential resource, in every problem they encounter they seek a solution and in every change they look for the new opportunity potential.  Entrepreneurs think internally: Rather than let others and external factors determine the value of outcomes, the entrepreneur uses their internal locus of control to direct their destiny, to work for their achievements, to delay gratification, and to plan with an eye for long-term benefits. Entrepreneurs never think of themselves as victims. Everything they do has a personal development focus, even when doing unappreciated work for others.  Entrepreneurs think of value-add: Rather than singularly employ a 'business-savvy' focus of extracting benefit by exploiting a market/customer weakness, entrepreneurs focus primarily on creating new value for others (by increased benefit or reduced cost) and then finding a way of securing a slice of that increased value for themselves. This Crazy-Quilt entrepreneurial principle seeks partnerships with committed people and organizations willing to jointly create the future and are least concerned with traditional competitive analyses and strategic planning approaches.  Entrepreneurs think of making meaning:
  • 4. Ch Muhammad Irfan +92-345-4426176 facebook.com/chmuhammedirfan skype id: ch.irfan786 Entrepreneurs have an inherent desire to make a difference and to make their world a better place. So, for the entrepreneur, making money becomes a means to that end and not an end in itself. As Richard Branson says: “Above all, you want to create something you are proud of ... I can honestly say that I have never gone into any business purely to make money … I’ve had to create companies that I believe in 100%. These are companies I feel will make a genuine difference.” Topic: Define entrepreneurship? Write a note on history of entrepreneurship. The activity of setting up a business or businesses, taking on financial risks in the hope of profit. The begging of Trade: The original entrepreneurs were, of course, traders and merchants. The first known instance of humans trading comes from New Guinea around 17,000 BCE, where locals exchanged obsidian, a black volcanic glass used to make hunting arrowheads for other needed goods. These early entrepreneurs exchanged one set of goods for another. Around 15,000 BCE, the first animal domestication began taking place, and around 10,000 BCE, the first domestication of plants. This step toward agriculture was critical for the advancement of the human species. Now, instead of having to continually move around as nomadic tribes, seeking new places to hunt and to gather, we could stay in one place. The First Cities The Middle East’s fertile crescent between the Tigris and the Euphrates had the right mix of plants and animals to sustain the foundations of civilization. Around 4,000 BCE, people in central Asia tamed horses, giving them a major advantage in both agricultural work and warfare. By 3,000 BCE, the first settlements and cities formed in Sumeria (modern day Iraq). During this timeframe, the city of Uruk along the banks of the Euphrates River was home to 50,000 people in an amount of space that would have previously supported just one hunter-gatherer. Humans had become much more efficient at generating the food and energy necessary to support their communities. Trade Routes Allow Ideas and Memes to Spread Trade routes between the new cities soon sprang up. Donkeys, horses, and camels enabled trade caravans between civilizations, moving both goods and ideas. Ships were built to carry trade over the seas. Networks and hubs soon formed and more complex structures emerged. Great Pyramids were built in Cairo. Temples were built in Sumeria. The Invention of Money Early trade consisted of barter (one good for another). If Tom had twenty cows and Igor had eighty hens, and Tom and Igor agreed that one cow was worth four hens, then the trade could take place. The problem with the barter system, however, was that in order for a trade to take place, both parties had to want what the other party had. This “co-incidence of wants” often did not happen. Thus, the demands of growing business and trade gave rise to a money system. Silver rings or bars are thought to have been used as money in Ancient Iraq before 2000 B.C. Early forms
  • 5. Ch Muhammad Irfan +92-345-4426176 facebook.com/chmuhammedirfan skype id: ch.irfan786 of money (called specie) would be often be commodities like seashells, tobacco leaves, large round rocks, or beads. The Creation of Markets With a population spurt starting around 1470, cities, markets, and the volume of trade grew. Banking, initially started by Ancient Mesopotamians, grew to new heights and complexities; the guild system expanded; and the idea that a business was an impersonal entity, with a separate identity from its owner, started to take hold. Silver imports from the new world drove expanded trade and bookkeepers created standardized principles for keeping track of a firm’s accounts based on Luca Pacioli’s accounting advances. Early entrepreneurs, called merchants and explorers, began to raise capital, take risks, and stimulate economic growth. Capitalism had begun. Define Entrepreneurship? Write a note on entrepreneurial process: The Entrepreneur is a change agent that acts as an industrialist and undertakes the risk associated with forming the business for commercial use. An entrepreneur has an unusual foresight to identify the potential demand for the goods and services. 1. Discovery: An entrepreneurial process begins with the idea generation, wherein the entrepreneur identifies and evaluates the business opportunities. The identification and the evaluation of opportunities is a difficult task; an entrepreneur seeks inputs from all the persons including employees, consumers, channel partners, technical people, etc. to reach to an optimum business opportunity. Once the opportunity has been decided upon, the next step is to evaluate it. An entrepreneur can evaluate the efficiency of an opportunity by continuously asking certain questions to himself, such as, whether the opportunity is worth investing in, is it sufficiently attractive, are the proposed solutions feasible, is there any competitive advantage, what are the risk associated with it. Above all, an entrepreneur must analyze his personal skills and hobbies, whether these coincides with the entrepreneurial goals or not. 2. Developing a Business Plan: Once the opportunity is identified, an entrepreneur needs to create a comprehensive business plan. A business plan is critical to the success of any new venture since it acts as a benchmark and the evaluation criteria to see if the organization is moving towards its set goals. An entrepreneur must dedicate his sufficient time towards its creation, the major components of a business plan are mission and vision statement, goals and objectives, capital requirement, a description of products and services, etc. 3. Resourcing: The third step in the entrepreneurial process is resourcing, wherein the entrepreneur identifies the sources from where the finance and the human resource can be arranged. Here, the entrepreneur finds the investors for its new venture and the personnel to carry out the business activities. 4. Managing the company: Once the funds are raised and the employees are hired, the next step is to initiate the business operations to achieve the set goals. First of all, an
  • 6. Ch Muhammad Irfan +92-345-4426176 facebook.com/chmuhammedirfan skype id: ch.irfan786 entrepreneur must decide the management structure or the hierarchy that is required to solve the operational problems when they arise. 5. Harvesting: The final step in the entrepreneurial process is harvesting wherein, an entrepreneur decides on the future prospects of the business, i.e. its growth and development. Here, the actual growth is compared against the planned growth and then the decision regarding the stability or the expansion of business operations is undertaken accordingly, by an entrepreneur. The entrepreneurial process is to be followed, again and again, whenever any new venture is taken up by an entrepreneur, therefore, it’s an ever ending process. Topic: As an entrepreneur how do you establish a Corporate Entrepreneurship in the organization: Corporate entrepreneurship is especially crucial for large companies, enabling these organizations - that are traditionally averse to risk-taking - to innovate, driving leaders and teams toward an increased level of corporate enterprising. In addition to the obvious benefits obtained through innovation, this approach also provides the organizational benefit of setting the stage for leadership continuity. Step one: Secure a commitment to corporate entrepreneurship in the organization by top, upper, and middle management levels. Establish initial framework and embrace the concept identify, select, and train Step two: Identify ideas and areas that top management is interested in supporting. Identify amount of risk money available to develop the concept. Establish overall program expectations and target results of each corporate venture. Establish mentor/sponsor system. Step three: Use of technology to ensure organizational flexibility. corporate entrepreneurs. Step four: Identify interested managers to train employees and share their experiences. Step five: Develop ways for the organization to get closer to its customers. Step six: Learn to be more productive with fewer resources. Step seven: Establish a strong support structure for corporate entrepreneurship. Step eight: Tie rewards to the performance of the entrepreneurial unit. Finally: Implement an evaluation system that allows successful entrepreneurial units to expand and unsuccessful ones to be eliminated.
  • 7. Ch Muhammad Irfan +92-345-4426176 facebook.com/chmuhammedirfan skype id: ch.irfan786 Topic: Discuss the various difference b/w managerial and entrepreneurial decision making: The difference between the entrepreneurial style and the managerial style (administrative domain) involves five business dimensions. A. Strategic Orientation. 1. The entrepreneur’s strategic orientation depends on his or her perception of the opportunity. 2. When the use of planning systems is the strategic orientation, the administrative domain is operant. C. Commitment to Opportunity. 1. The entrepreneurial domain is pressured by the need for action and has a short time span in terms of opportunity commitment. 2. The administrative domain is not only slow to act on an opportunity, but the commitment is usually for a longer time span. D. Commitment of Resources. 1. An entrepreneur is used to having resources committed at periodic intervals, often based on certain tasks or objectives being reached. 2. In acquiring these resources the entrepreneur is forced to maximize resource use. 3. In the administrative domain, the commitment of resources is for the total amount needed. 4. Administrative-oriented individuals receive personal rewards by effectively administering the resources under their control. E. Control of Resources. 1. The administrator is rewarded by effective resource administration and has a drive to own or accumulate as many resources as possible. 2. The entrepreneur, under pressure of limited resources, strives to rent resources on an as- needed basis. F. Managerial Structure. 1. In the administrative domain, the organizational structure is formalized and hierarchical in nature. 2. The entrepreneur employs a flat organizational structure with informal networks. Topic: Define new entry. How is a new entry opportunity generated?
  • 8. Ch Muhammad Irfan +92-345-4426176 facebook.com/chmuhammedirfan skype id: ch.irfan786 Offering a new product to an established or new market.  Resources as a Source of Competitive Advantage Resources are the basic building blocks to a firm’s functioning and performance; the inputs into the production process. i. They can be combined in different ways. ii. A bundle of resources provides a firm its capacity to achieve superior performance. Resources must be: i. Valuable. ii. Rare. iii. Unique.  Creating a Resource Bundle That Is Valuable, Rare, and Inimitable i. Entrepreneurs need to draw from their unique experiences and knowledge. ii. Market knowledge - Information, technology, know-how, and skills that provide insight into a market and its customers. iii. Technological knowledge - Information, technology, know-how, and skills that provide insight into ways to create new knowledge.  Assessing the Attractiveness of a New Entry Opportunity Depends on the level of information and the willingness to make a decision without perfect information.  Information on a New Entry i. Prior knowledge and information search ii. More knowledge ensures a more efficient search process. iii. Search costs include time and money. iv. The viability of a new entry can be described in terms of a window of opportunity.  Comfort with Making a Decision under Uncertainty i. The trade-off between more information and the likelihood that the window of opportunity will close provides a dilemma for entrepreneurs. ii. Error of commission - Negative outcome from acting on the perceived opportunity. iii. Error of omission - Negative outcome from not acting on the new entry opportunity. Topic: Discuss the various risk reduction strategies: Select a business structure that limits personal liability. Change your business structure from a sole proprietorship in which you are personally liable for business operations to a corporation or limited liability company where you have limited liability. Transfer risk to insurance companies by insuring against major risks Transfer risk to insurance companies by insuring against major risks such as damage to your facilities, product liability, injuries to customers or suppliers and death or incapacity of company principals. Perform a risk analysis by evaluating the consequences of risky activities
  • 9. Ch Muhammad Irfan +92-345-4426176 facebook.com/chmuhammedirfan skype id: ch.irfan786 Perform a risk analysis by evaluating the consequences of risky activities, the likelihood of the consequences occurring and the benefits of the risky activities. Avoid risk by not carrying out activities that have severe and likely consequences and low benefits. Transfer the risk of activities Transfer the risk of activities with severe and likely consequences but high benefits to other parties. Create a new, independent company to carry out these activities or assign them to suppliers or partners. Reduce risk from product failure and warranty claims Reduce risk from product failure and warranty claims by implementing a quality assurance program. Develop a system of reporting from customer service to identify problems. Structure the quality assurance program to document production tasks and product testing. Link the problems reported by customer service to specific failures in production or testing procedures and institute corrective action. Reduce risk of surprises in operating results Reduce risk of surprises in operating results by keeping accurate records and instituting effective controls. Put in place a system that limits who can authorize specific actions and how much they can spend. Implement a reporting system that gives you key information about company performance. Evaluate the controls and reporting system by comparing actual practice and performance to the control procedures and the reported information. Reduce financial risk by managing your accounts receivable Reduce financial risk by managing your accounts receivable to minimize outstanding balances and identify poor credit risks. Implement credit and payment standards, specifying which credit scores and payment records are acceptable. Evaluate customer payments and ask for advance payment from customers who don't meet the standards. Reduce financial risk by keeping outstanding loans and financing needs to a minimum Control growth at a rate that the company can finance internally. If the company can't pay off some loans, replace short-term credit with long-term, fixed-rate loans. Topic: Explain the importance of entrepreneurial background and characteristics: ENTREPRENEURIAL BACKGROUND Only a few background characteristics have differentiated the entrepreneur from the general populace or managers. Childhood Family Environment The impact of birth order and social status has had conflicting research results. There is strong evidence that entrepreneurs, both male and female, tend to have self-employed or entrepreneurial fathers. Having a father who is self-employed provides a strong inspiration in the example of independence and flexibility of self-employment. This feeling of independence is often further enforced by an entrepreneurial mother. Education
  • 10. Ch Muhammad Irfan +92-345-4426176 facebook.com/chmuhammedirfan skype id: ch.irfan786 1. Education appears important in the upbringing of the entrepreneur, in the level of education obtained and in playing a major role in coping with problems. 2. Although formal education is not necessary for starting a new business, it does provide a good background. 3. In education, entrepreneurs experienced some disadvantage, with few having degrees in engineering, science, or math. 4. The ability to deal with people and communicate clearly in written and spoken work is also important. Age  Entrepreneurial age is the age of the entrepreneur reflected in the experience.  Entrepreneurial experience is one of the best predictors of success.  In chronological age, most entrepreneurs start their careers between ages 22 and 55.  Earlier starts in an entrepreneurial career seem to be better than later ones.  Generally, male entrepreneurs start their first venture in their early 30s, while women tend to do so in their middle 30s.  Between 30-50 years’ old Work History 1. Dissatisfaction with one’s job often motivates the launching of a new venture. 2. (Gift within trouble) 3. Previous technical and industry experience is also important once the decision to start a business is made. 4. Experience in the following areas is particularly important: financing; product or service development; manufacturing; development of distribution channels; and preparation of a marketing plan. 5. As the venture becomes established, managerial experience and skills become more important. 6. Entrepreneurial experience becomes increasingly important as the complexity of the venture increases. MOTIVATION 1) While motivations may vary, the reason cited most often for becoming an entrepreneur independence-not wants to work for anyone else. 2) Other motivating factors differ between male and female entrepreneurs. 3) Money is the second reason for men’s starting a venture. 4) Job satisfaction, achievement, opportunity, and money are the second order reasons for women. Entrepreneurial Characteristics: 1. Key Personal Attributes 2. Strong Managerial Competencies 3. Good Technical Skills Topic: Identify and define the methods of Generating new ideas:
  • 11. Ch Muhammad Irfan +92-345-4426176 facebook.com/chmuhammedirfan skype id: ch.irfan786 Focus Groups A focus group is defined as a group of individuals providing information in a structured format. A moderator leads the group of about 8 to 14 participants through an open, in-depth discussion rather than simply asking questions to solicit participants’ responses. For a new product or services area, the moderator focuses the discussion of the group in either a directive or non-directive manner. The group is stimulated by comments from other group members in creatively conceptualizing and developing a new product or service to fill a market need. The focus group is an excellent approach for initially screening ideas and concepts. Existing company can use this method to expand a section or department to be able to achieve greater productivity in its services. Brainstorming The brainstorming method allows a group of individual to be stimulated to greater creativity by interacting with each other and participating in organized group experience. Most of the ideas generated from the group have no basis for further development, yet there are times that a good idea emerges. This has a greater frequency of occurrence when the brainstorming effort focuses on a specific product or market area. For this approach to be successful there should be no negative comment or criticism, quantity of ideas should be encouraged, the wider the idea, the better and combinations and improvements of ideas are encouraged. Many modern commercial banks successfully used brainstorming techniques to develop a series of product for their clients or customers. Problem Inventory Analysis Problem inventory analysis is a method for obtaining new ideas and solutions by focusing on existing problems. In this approach, the customers or consumers are provided with a list of problems in a general product category. Thereafter, they are asked to identify and discuss product in each category that have a particular problem. This method is effective when an improved service/product is desired. When known product or services are related to suggested problems, a new product idea emerges. Result from product inventory analysis must be carefully evaluated as they may actually reflect a new business opportunity. For maximal result, it is advisable that problem inventory analysis should be used primarily to identify new product ideas from existing product before further evaluation. Topic: Discuss the seven creative problems solving techniques: Find the Right Problems to Solve Too often our approach to problem solving is reactive, we wait for the problems to arise. Firstly, in our 7 problem solving steps, we advocate taking a proactive approach, go and find problems to solve; important and valuable problems. The real starting point then for any problem solving process is to find the right problem to solve.  Define the Problem
  • 12. Ch Muhammad Irfan +92-345-4426176 facebook.com/chmuhammedirfan skype id: ch.irfan786 It is very tempting to gloss over this step and move to analysis and solutions. However, like the first step, it is one of the secrets of effective problem solving. Combining problems that are valuable to solve, with defining exactly what you are trying to solve, can dramatically improve the effectiveness of the problem solving process. The secret to defining the problem, is really about attitude. Try to see every problem as an opportunity.  Analyze the Problem Analysis is a process of discovery of the facts, finding out what you know about the situation. In doing so you are breaking down complexity, stripping away the superficial and getting to the causes/issues.  Develop Opportunities There are always more than one way to solve a problem, so take time to develop plenty of creative possibilities to solve the problem.  Select the Best Solution Selecting is about making choices, about deciding. To do this you need to weigh up the competing value and risk of the different options you generated in the previous step.  Implement Good solutions are often only as good as the way they are implemented. Implementation requires project management and a determination to deliver the outcomes essential to solving the problem you originally defined.  Evaluate and Learn You will have done some things really well through this seven step problem solving process. It would be all too easy to forget them in rushing to solve the next problem, or to implement the solution. You should evaluate at least two areas: How you carried out the seven step problem solving process The effectiveness of the solution you implemented. Did it deliver the outcomes you expected? Topic: Differentiate between domestics and international entrepreneurship? International Entrepreneurship: Global entrepreneurship is a trait or characteristic in our definition of a global leader. Global entrepreneurs are professionals who use their global understanding and connections to identify transnational and cross-cultural opportunities and turn them into new value-creating initiative. Doing business internationally is not the same as doing business at home. There are new skills to learn and new knowledge to acquire about the country you will be going into. You will need to learn about the different laws and regulations, the different customer buying habits, and change your marketing strategies and materials to appeal to the new country you are entering. Cultures No two cultures are the same and understanding both the social and business culture in another country is the first key to success. Culture defines everything a society does, from its business
  • 13. Ch Muhammad Irfan +92-345-4426176 facebook.com/chmuhammedirfan skype id: ch.irfan786 practices, to its response to advertising and marketing, to negotiating sales. It is important to include research on the culture of the country(s) that you intend to sell to prior to entering their market. Understanding these, often sensitive, areas will mean that you are better prepared when first entering the market. Level of Competition The level of competition you will experience in foreign markets is likely to be more dynamic and complex than you experience in domestic markets. Market Intelligence The key points to determine when gathering market intelligence on the market you intend to enter are:  Understanding how the market works  Who your direct competition is, and  The best market entry strategy Politics/Government/Legal Systems No two countries have the same political and legal systems. Each government has its own policies relating to foreign firms and products. The key is to understand that once you are in a foreign market you must abide by the rules and laws of that country, not the ones in your own market. International Law Countries determine their laws based on the needs of their citizens not the concerns of foreign companies. By and large, international law is a gentlemen's agreement which is honored, but not always. Conclusion: There are a many differences between domestic and international business. Whether they are cultural, technical or legal they require an understanding and an appreciation of the differences. Following are a few web links to help you get started. Topic: Explain the sources of new product development ideas for entrepreneurs with examples:  Internal Sources: Internal sources are the great way to find new ideas. Internal sources can be divided into two parts. They are as follows- 1. R & D (Research and Development): It is the formal department of any organization to generate new ideas. R&D department research according to the company’s future plan and then come up with the new ideas which complete its journey with the commercialization of the idea (product). One recent survey showed that traditional R&D only contribute less than 15% of the ideas of the organization. However, the importance of R&D in the organization is not only depends on the idea generation, they also do some other important things. 2. Employees: Companies can use the brain of their employees. If customers are the Oxygen of any company, then employees are the heart of that company. All level of employees from executive to top management can be the great source of ideas. One
  • 14. Ch Muhammad Irfan +92-345-4426176 facebook.com/chmuhammedirfan skype id: ch.irfan786 recent research showed that almost 45% of the ideas comes from the employees. Many companies now use web technology to get the ideas from their internal employees. In that web form employees can share their ideas about new product. However, picking up the great idea from it and rewarding the employees can encourage your employees to be more creative and contribute more in future.  External Sources: Companies can also gain good new product development ideas from external sources. External sources are those which is affiliated with the company externally. Some important external sources for idea generation is discussing below. 3. Customers: Most probably customers are the most important sources to get new product development ideas. Customer knows best what they need and what they are looking for. It is the most important thing to deliver satisfaction by providing exactly what your customers want. For instances, when you know that your customer need a specific product or a special feature on any particular product then it will be easier to make that exactly what your customer need and then you will get satisfied customers. Thus way you can build long term relationship. 4. Distributors and Suppliers: Distributors works very closely with the market and they know consumer problems and their need. Distributors can give the ideas for new product possibilities. Suppliers can also help with the information of the market like new concept, technique or materials which can be used for developing new products. 5. Competitors: Competitors are another important source. One can analyze their competitors and can find many things which can be used for idea generation. Researching competitors can give you the idea that which thing you are missing or which thing they are missing, you can decide then what things you need to include in your new product. Remember your competitors are not your enemy, they are your strength. 6. Others: Other idea sources includes outside Consultancies, Design Firms and Online Communities, Trade Magazines, Shows and Seminars, Government agencies, Advertising agencies, Marketing research firms, Universities, Commercial laboratories, Inventories and so on. Topic: Write a note on product planning and development process: New Product Development: Developing a new product involves a number of stages which typically center on the following key areas:  The Idea: Every product has to start with an idea. In some cases, this might be fairly simple, basing the new product on something similar that already exists. In other cases, it may be something revolutionary and unique, which may mean the idea generation part of the process is much more involved. In fact, many of the leading manufacturers will have whole departments that focus solely on the task of coming up with ‘the next big thing’.  Research:
  • 15. Ch Muhammad Irfan +92-345-4426176 facebook.com/chmuhammedirfan skype id: ch.irfan786 An organization may have plenty of ideas for a new product, but once it has selected the best of them, the next step is to start researching the market. This enables them to see if there’s likely to be a demand for this type of product, and also what specific features need to be developed in order to best meet the needs of this potential market.  Development: The next stage is the development of the product. Prototypes may be modified through various design and manufacturing stages in order to come up with a finished product that consumers will want to buy.  Testing: Before most products are launched and the manufacturer spends a large amount of money on production and promotion, most companies will test their new product with a small group of actual consumers. This helps to make sure that they have a viable product that will be profitable, and that there are no changes that need to be made before it’s launched.  Analysis: Looking at the feedback from consumer testing enables the manufacturer to make any necessary changes to the product, and also decide how they are going to launch it to the market. With information from real consumers, they will be able to make a number of strategic decisions that will be crucial to the product’s success, including what price to sell at and how the product will be marketed.  Introduction: Finally, when a product has made it all the way through the new product development stage, the only thing left to do is introduce it to the market. Once this is done, good product life cycle management will ensure the manufacturer makes the most of all their effort and investment. Topic: Identify and define the various Entry Modes of international entrepreneurs: The decision of how to enter a foreign market can have a significant impact on the results. Expansion into foreign markets can be achieved via the following four mechanisms: Exporting Exporting is the marketing and direct sale of domestically produced goods in another country. Exporting is a traditional and well established method of reaching foreign markets. Since exporting does not require that the goods be produced in the target country, no investment in foreign production facilities is required. Most of the costs associated with exporting take the form of marketing expenses.  Licensing
  • 16. Ch Muhammad Irfan +92-345-4426176 facebook.com/chmuhammedirfan skype id: ch.irfan786 Licensing essentially permits a company in the target country to use the property of the licensor. Such property usually is intangible, such as trademarks, patents, and production techniques. The licensee pays a fee in exchange for the rights to use the intangible property and possibly for technical assistance. Joint Venture There are five common objectives in a joint venture: market entry, risk/reward sharing, technology sharing and joint product development, and conforming to government regulations. Other benefits include political connections and distribution channel access that may depend on relationships. Foreign Direct Investment Foreign direct investment (FDI) is the direct ownership of facilities in the target country. It involves the transfer of resources including capital, technology, and personnel. Direct foreign investment may be made through the acquisition of an existing entity or the establishment of a new enterprise. Topic: What are the steps in registering your trademark of your business? Procedure/Steps for Trademark Registration  Filing of an application for registration by a person claiming to be the proprietor of a trademark, in the office of the Trade Mark Registry, within the territorial limits of the place of business in India.  Examination of the application by the Registrar to ascertain whether it is distinctive and does not conflict with existing registered or pending trademarks and examination report is issued.  Publication of the application after or before acceptance of the application in the Trademark Journal.  After publication if any person gives notice of his opposition to the registration within three months which may be extended to the maximum of one month.  If the opposition has been decided in favor of the applicant of the registration of trademark, the Registrar shall register the Trademark.  On the registration of the Trademark the Registrar shall issue to the applicant a Trademark Registration Certificate. Topic: What is business plan? Write down all business segment for a business plan: Definition: A written document describing the nature of the business, the sales and marketing strategy, and the financial background, and containing a projected profit and loss statement.  Title Page and Contents A business plan should be presented in a binder with a cover listing the name of the business, the name(s) of the principal(s), address, phone number, e-mail and website
  • 17. Ch Muhammad Irfan +92-345-4426176 facebook.com/chmuhammedirfan skype id: ch.irfan786 addresses, and the date. You don't have to spend a lot of money on a fancy binder or cover. Your readers want a plan that looks professional, is easy to read and is well-put- together.  Executive Summary The executive summary, or statement of purpose, succinctly encapsulates your reason for writing the business plan. It tells the reader what you want and why, right up front. Are you looking for a $10,000 loan to remodel and refurbish your factory? A loan of $25,000 to expand your product line or buy new equipment? How will you repay your loan, and over what term? Would you like to find a partner to whom you'd sell 25 percent of the business? What's in it for him or her? The questions that pertain to your situation should be addressed here clearly and succinctly. Description of the Business The business description usually begins with a short explanation of the industry. When describing the industry, discuss what's going on now as well as the outlook for the future. Do the necessary research so you can provide information on all the various markets within the industry, including references to new products or developments that could benefit or hinder your business. Base your observations on reliable data and be sure to footnote and cite your sources of information when necessary. Remember that bankers and investors want to know hard facts--they won't risk money on assumptions or conjecture. Description of the Product or Service The business description can be a few paragraphs to a few pages in length, depending on the complexity of your plan. If your plan isn't too complicated, keep your business description short, describing the industry in one paragraph, the product in another, and the business and its success factors in two or three more paragraphs.  Market Analysis A thorough market analysis will help you define your prospects as well as help you establish pricing, distribution, and promotional strategies that will allow your company to be successful vis-à-vis your competition, both in the short and long term.  Competitive Analysis The purpose of the competitive analysis is to determine:  the strengths and weaknesses of the competitors within your market.  strategies that will provide you with a distinct advantage.  barriers that can be developed to prevent competition from entering your market.  any weaknesses that can be exploited in the product development cycle.  Operations and Management The operations and management component of your plan is designed to describe how the business functions on a continuing basis. The operations plan highlights the logistics of the organization, such as the responsibilities of the management team, the tasks assigned to each division within the company, and capital and expense requirements related to the operations of the business.
  • 18. Ch Muhammad Irfan +92-345-4426176 facebook.com/chmuhammedirfan skype id: ch.irfan786  Financial Components of Your Business Plan After defining the product, market and operations, the next area to turn your attention to are the three financial statements that form the backbone of your business plan: the income statement, cash flow statement, and balance sheet. Topic: Write a note on Followings: Patents: Patent protect functional expressions of an idea – not the idea itself. A machines, method/process, manufacture, compositions of matter, and improvements of any of these items can be patented. Thus, I can patent a design for the nozzle on a rocket, or the method of making the rocket, or the method of making the rocket fuel, or the metal in which the rocket fuel is stored, or a new way of transporting the rocket fuel to the rocket. But I cannot patent the broad “idea” of a rocket. Copyrights: Copy Rights protect the specific creative expression of an idea through any medium of artistic/creative expression – i.e., paintings, photographs, sculpture, writings, software, etc. A copyright protects your painting of a haystack, but it would not prohibit another painter from expressing their artistry or viewpoint by also painting a haystack. Likewise, while Ian Fleming was able to receive a copyright on his particular expression of the idea of a secret agent (i.e., a debonair English secret agent), he could not prevent Rich Wilkes from receiving a copyright on his expression of the idea of a secret agent (i.e., a tattooed bald extreme athlete turned reluctant secret agent). Trademarks: Trade marks protect any symbol that indicates the source or origin of the goods or services to which it is affixed. While a trademark can be extremely valuable to its owner, the ultimate purpose of a trademark is to protect consumers – that is, the function of a trademark is to inform the consumer where the goods or services originate. The consumer, knowing the origin of the goods, can make purchasing decisions based on prior knowledge, reputation or marketing. Trade Secrets: Broadly speaking, any confidential business information which provides an enterprise a competitive edge may be considered a trade secret. Trade secrets encompass manufacturing or industrial secrets and commercial secrets. The unauthorized use of such information by persons other than the holder is regarded as an unfair practice and a violation of the trade secret. Depending on the legal system, the protection of trade secrets forms part of the general concept of protection against unfair competition or is based on specific provisions or case law on the protection of confidential information. Topic: Write down the outline of your business plan: Executive summary
  • 19. Ch Muhammad Irfan +92-345-4426176 facebook.com/chmuhammedirfan skype id: ch.irfan786 Begin your plan with an executive summary, which you should write last. The summary condenses your main points into two or three pages so investors can quickly decide if the opportunity suits them enough to keep reading. Product What are you selling? Describe your product, giving enough information to imagine the product. Eliminate the technical details-if you can't get the idea across quickly, you haven't thought it through enough. Marketing Who buys, why and how? Who are your customers? What need are you filling? Give numbers and sources to give a feel for the opportunity size. For example, "Tractors Quarterly, May 2000, says 20,000 tractor manufacturers need 15 hours to change production lines. Our patent-protected TractorGizmoT reduces that to 12 seconds, saving an estimated $55 billion yearly." Business model Show where the money comes from. Gillette loses money on razors, but makes money-lots of money-selling blades. Discuss who pays you, how much they pay and how often The team Introduce your team and advisory board. Good ideas are common; talented managers aren't. Show you have the right team for the idea by highlighting the relevant background of your team members. If you make ball bearings, emphasize your experience in manufacturing, not your time as a college newspaper editor. Competition What's your competition and why will you win? Competition may not be a company. Personal checkbook program Quicken considered pencil and paper, not other computer programs, the biggest competition. Financing How much money do you have? Where did it come from? How much do you want? How will you use it? What return are you promising investors? Operations Eventually you'll need to get things done. Describe how you'll deliver your product, the problems you expect and how you'll master them. If your business allows 100,000 people a year to order custom-made suits by telephone, here's where you say how you'll actually make and deliver 100,000 suits. Scale How will you grow? A diner isn't run the same way as a 250-table restaurant. Explain your growth plans and how your idea scales. Development How are you doing so far? If you're already under way, talk about your progress. In any event, lay out the next few months' milestones so you and your investors can track your progress. Topic:
  • 20. Ch Muhammad Irfan +92-345-4426176 facebook.com/chmuhammedirfan skype id: ch.irfan786 List all methods of generating new ideas for the entrepreneurial venture and explain any three of them: METHODS OF GENERATING NEW IDEAS Even with the wide variety of sources available, coming up with an idea to serve as the basis for the new venture can still be a difficult problem. The entrepreneur can use several methods to help generate and test new ideas, including focus groups, brain storming and problem inventory analysis. Focus groups Group of individuals providing information in a structured format is called a focus group. The group of 8 to 14 participants is simulated by comments from other group members in creatively conceptualizing and developing new product idea to fulfill a market need. Brainstorming A group method of obtaining new ideas and solutions is called brainstorming. The brainstorming method for generating new ideas is based on the fact that people can be stimulated to greater creativity by meeting with others an d participating with organized group experiences. Although most of the ideas generated from the group have no basis for further development, often a good idea emerges. Problem inventory analysis Problem inventory analysis uses individuals in a manner that is analogous to focus groups to generate new product ideas. However instead of generating new ideas themselves, consumers are provided with a list of problems in a general product category. They are then asked to identify and discuss products in this category that have the particular problem. This method is often effective since it is easier to relate known products to suggested problems and arrive at a new product idea then to generate an entirely new idea by itself. Topic: What are the characteristics of a Marketing plan? You have a product people love and you understand who the people are that love it. Now would be a great time to build a marketing plan. The plan will change and evolve over time but having a structure helps to make sure that your marketing spend is aligning with your goals without being completely ad-hoc, spray and pray, or flavor of the day. The plan shouldn’t be a massive undertaking – it’s a rough guide that documents what you are planning on doing and why you are doing it. Good plans should contain the following: Segmentation – At this stage you should understand what segments love you offering and why. If you don’t, you probably aren’t ready to build a marketing plan yet and you should go back and focus on finding a fit between your offering and a set of markets. This is where you document with as much detail as humanly possible the segments that you are going to target. Who are the groups, what are their characteristics and how do you identify them?
  • 21. Ch Muhammad Irfan +92-345-4426176 facebook.com/chmuhammedirfan skype id: ch.irfan786 Competitive Alternatives – This is a documented list of what your customers would consider alternatives to your product or service. Don’t go crazy on this documenting every little bit of functionality. The purpose of this section is to help you articulate what really differentiates your offering from others in the space. These are generally macro things and not niggly detailed nuances of features. Differentiated Points of Value (by segment) – For each segment, what are the top 3 or 4 differentiators that your offering has versus others. Remember this isn’t just about technology or features. It often includes things like pricing, delivery options, ease of use, time to value, etc. Messaging and Positioning (by segment) – Working from the previous section create a set of messages for each segment. This should be a succinct set of no more than 3/4 messages that get across your key points. (For more on this see my post on Crafting Simple Value Statements) Marketing Goals and Measures – What are the goals of your marketing plan and what metrics are you tracking that are associated with those goals? For example, you might decide that increasing customer acquisition by x% is a key goal. You can tie acquisition to site visits, product signups, email or blog signups, abandon rates, etc. Tactical Plan, Budget, Owners – Based on the above goals and measures this is the set of marketing plans you plan on executing to drive those results and the costs associated with each of those tactics. The tactics are broken into discreet items of work (i.e. for example a tactic such as a webinar will include creating the invite list, writing/designing the mailer, sending the invite, creating the webinar content, etc.) and assigned to an owner. Timeline– The tactics need to be broken into work items and plotted on a timeline so they can be tracked on a regular (for my teams it’s usually weekly) basis. OR Eight parameters can help in gauging the viability of strategic marketing plans. If your planning process reflects these attributes, it's probably off to a good start.  Accountability—responsibility is assigned for successful completion of initiatives.  Balance—the plans inform and guide not only financial decision-making, but also operational and human resource issues.  Flexibility—a mechanism for changing and updating the plan is built into the process.
  • 22. Ch Muhammad Irfan +92-345-4426176 facebook.com/chmuhammedirfan skype id: ch.irfan786  Manageability—in-process measures are identified to ensure processes are working as intended, critical performance issues are addressed, resources required are projected, and methods of status reporting are in place.  Prioritization—priorities are established whenever multiple interdependent actions have been planned.  Realism—the question of what the organization really can do versus what it would like to do is addressed rationally, though the tone may be optimistic.  Specificity—expected results and milestones are clearly defined, along with the specific actions for implementation and the deliverables for each step.  Sustainability—a sufficient time period is allowed to close performance gaps. Topic: Write a note on steps in marketing research: Step 1. Define the Objective & Your “Problem” Perhaps the most important step in the market research process is defining the goals of the project. At the core of this is understanding the root question that needs to be informed by market research. There is typically a key business problem (or opportunity) that needs to be acted upon, but there is a lack of information to make that decision comfortably; the job of a market researcher is to inform that decision with solid data. Step 2. Determine Your “Research Design” Now that you know your research object, it is time to plan out the type of research that will best obtain the necessary data. Think of the “research design” as your detailed plan of attack. In this step you will first determine your market research method (will it be a survey, focus group, etc.?). You will also think through specifics about how you will identify and choose your sample (who are we going after? where will we find them? how will we incentivize them? etc.). Step 3. Design & Prepare Your “Research Instrument” In this step of the market research process, it’s time to design your research tool. If a survey is the most appropriate tool (as determined in step 2), you’ll begin by writing your questions and designing your questionnaire. If a focus group is your instrument of choice, you’ll start preparing questions and materials for the moderator. You get the idea. This is the part of the process where you start executing your plan. Step 4. Collect Your Data This is the meat and potatoes of your project; the time when you are administering your survey, running your focus groups, conducting your interviews, implementing your field test, etc. The answers, choices, and observations are all being collected and recorded, usually in spreadsheet form. Each nugget of information is precious and will be part of the masterful conclusions you will soon draw.
  • 23. Ch Muhammad Irfan +92-345-4426176 facebook.com/chmuhammedirfan skype id: ch.irfan786 Step 5. Analyze Your Data Step 4 (data collection) has drawn to a close and you have heaps of raw data sitting in your lap. If it’s on scraps of paper, you’ll probably need to get it in spreadsheet form for further analysis. If it’s already in spreadsheet form, it’s time to make sure you’ve got it structured properly. Once that’s all done, the fun begins. Step 6. Visualize Your Data and Communicate Results You’ve spent hours poring through your raw data, building useful summary tables, charts and graphs. Now is the time to compile the most meaningful take-away into a digestible report or presentation. Topic: Identify and define the various Legal Forms of Business which one you prefer the best why: Here are a range of legal structures associated with different forms of business: Sole Proprietorship / Sole Traders: Sole trader businesses are the easiest to set up because there is no complex paperwork. The business and the owner are the same person in law. The sole trader does not have limited liability which means that they are responsible for all the debts of the business. The sole trader has to produce an annual accounting return for the Inland Revenue. Partnerships: Partnerships are set up by a Deed of Partnership which is a document made out by the partners and witnessed by a solicitor. This Deed sets out the legal relationship between partners e.g. how profits will be shared out, responsibilities of partners etc. In traditional partnerships the partners had unlimited liability i.e. they were jointly responsible for the debts of their partnership. In 2001 this has been altered so that some large partnerships e.g. accountancy firms can have limited liability. Companies: Companies are separate in law from the individual owners (shareholders) of the business. This means that should the business run up debts the shareholders are only liable for these debts up to the sum they have contributed to the company. A number of Companies Acts have been passed setting out ways in which companies should conduct their affairs. Which one Is Prefer: For many small businesses, the best initial choice is either a sole proprietorship or, if more than one owner is involved, a partnership. Either of these structures makes good sense in a business where personal liability isn't a big worry -- for example, a small service business in which you are unlikely to be sued and for which you won't be borrowing much money. Sole proprietorships and partnerships are relatively simple and inexpensive to establish and maintain. Topic:
  • 24. Ch Muhammad Irfan +92-345-4426176 facebook.com/chmuhammedirfan skype id: ch.irfan786 How would you build a management team and culture for your business organization? Building a strong culture within a team is at the core of business success. You want a culture that recognizes and embraces shared values, attitudes, standards, and beliefs that characterize the goals of the organization. And it’s a good idea to make sure it suits the best people who work at the company while making a positive impression on customers and anyone else associated with the business. Following are some things I believe to be the cornerstones of a solid business culture. Transparency: At my company, we go over all the key metrics of the business with the entire company. The goal is for all employees to feel they know the thinking, responsibilities, and strategy at various levels of the company and can share ideas and feedback no matter who they are. Time to disconnect. We all need to hit the reset button once in a while -- people can't come in early and leave late every single day without getting burnt out at some level. While you want employees to have a work-hard founder’s mentality, you need to recognize the work-life integration that exists and how significant it is to make sure you have personally fulfilled, clear-thinking people. It’s important to understand that sometimes life will get in the way of business and everyone should be allowed to take care of pressing personal matters. Empowerment and a sense of freedom. You empower people by not micromanaging, erring on the side of giving people general guidelines rather than explicit, detailed directions. Informed employees are more involved and empowered in a company. And the more freedom people have to take on tasks, manage them, find solutions, and execute them, the more they feel connected to and woven into the company’s culture.  Physical space. If you haven’t watched Susan Cane’s TED Talk on introverts, I highly recommend it. She opened my mind to the idea that American businesses are built for extroverts, down to the floor plans of our office spaces. Although open spaces are great for some, other people need to be able to close the door to be at their most productive. It’s important to consider the comfort level of your employees before you decide how to lay out space or what type of office space to lease. Talking to customers and employees. At different points in a company’s maturation process, you're almost guaranteed to go through weeks or even months where you feel lost. When you haven’t nailed a product market fit or you’re having challenges relating to your product or corporate vision, the natural tendency is to turn your attention to where you or your team went wrong. Another way to try to solve these problems is by talking to your customers. Your organizational design. Simply put, organizational design is the processes, structure, and hierarchy you put into place that allow you to put your culture into practice. It’s “how you do things.” This will include your
  • 25. Ch Muhammad Irfan +92-345-4426176 facebook.com/chmuhammedirfan skype id: ch.irfan786 communication, company policies, team building, performance indicators, performance evaluations, division of responsibilities, and even how you schedule, and run, meetings. Topic: Discuss the advantages and disadvantages: An S corporation protects the personal assets of its shareholders. ... In a sole proprietorship or general partnership, owners and the business are legally considered the same—leaving personal assets vulnerable. Pass-through taxation. An S corporation does not pay federal taxes at the corporate level. Following are the advantages and disadvantages of S corporation:  Eliminating double taxation: In an S corporation, profits and losses are passed through to shareholders, and taxes are only paid once. Check with your state to see how it handles S Corporations. Some states do not recognize S Corporations and will tax such businesses as a regular C Corporation. Some states charge S Corporations a state tax, although the corporation will not have to pay federal tax.  Protection from liability: As the owner of an S Corporation, your personal assets are separate from the business’s assets and are therefore protected in case any judgments occur against the business.  More room for investors: S Corporations can have up to 100 shareholders.  Easier accounting rules: S Corporations without any inventory can use the cash method of accounting, which is much simpler than the accrual method. Check with your accountant about which option makes sense for your business. Here are some disadvantages of forming an S Corporation:  Rules and fees: Like a C Corporation, S Corporations are required to file a number of official state and federal documents, including Articles of Incorporation and corporate minutes. They must also hold regular shareholder meetings and pay the required government fees.  Shareholder restrictions: Realize that if an S Corporation has shareholders, the shareholders will be taxed for any income the company has, even if they did not receive any portion of that income. (In a C Corporation, shareholders are taxed only if they receive dividends.) In addition, S Corporations are only allowed to issue one class of stock, which may discourage some investors.  Salary requirements: The Internal Revenue Service requires all officers and owners of an S Corporation to make a salary, even if the company is not yet making a profit. This could be problematic for new businesses struggling to make payroll. A “reasonable salary” is what a person with the appropriate skills needed for the position would be paid on the free market. Topic:
  • 26. Ch Muhammad Irfan +92-345-4426176 facebook.com/chmuhammedirfan skype id: ch.irfan786 Briefly explain growth strategies based upon knowledge of product and or market: Growing a company takes a lot more than good intentions. You need a plan. With dozens of strategies to choose from, here are four that could put your company on the fast track to growth. Not every growth strategy is appropriate for every small business. The key to finding the right growth strategy is properly matching it to your company and its specific marketplace. Since the wrong strategy can devastate your business, it's important to determine whether you are selling new or emerging products in a new or existing market. Diversification (New Products/New Market) Diversification is a high-risk growth strategy, largely because both the products and the market are unproven territory for the entrepreneur. Though trailblazing emerging products and markets can be exhilarating, it can also be terrifying given the fact that neither you nor anyone else can rely on prior experience for reassurance. But if innovation is one of your company's defining characteristics, a diversification strategy will eventually become second nature. To achieve growth, you will need to be realistic about the risks you face and crystal clear about what you hope to achieve. Market Development (Existing Products/New Market) A more common scenario is one in which a small business owner attempts to develop a new market for their existing products and services. The new market can be geographical (e.g. foreign export) or an untapped segment of a domestic market. It's even possible to develop a new market for existing products by adjusting the product's packaging or expanding the product's distribution channels. In any event, a market development growth strategy requires a working knowledge of existing markets and the ability to gaps in the marketplace that can be exploited to your advantage. If your marketing skills are not up to the task, you will need the assistance of a skilled marketing professional to achieve growth in your new market. Product Development (New Products/Existing Market) A growth strategy based on product development is the mirror image of a market development strategy. Instead of pioneering a new market with existing products, you attempt to roll out a new product(s) in a market with which you are already familiar. Many small business owners are more comfortable working in this kind of scenario because they already possess an awareness of prevailing market conditions. However, a product development strategy can be just as challenging as a market development strategy because it often requires the business to develop new abilities and continuously adapt the products until they achieve marketplace success. Market Penetration (Existing Products/Existing Markets) Businesses that find themselves in a situation that involves neither new markets nor new products are forced to grow through a market penetration strategy, a strategy that is designed to give the business a greater percentage of market share. This type of strategy usually seeks to gain a competitive edge through pricing, marketing, or other initiatives. Additionally, market penetration can be achieved by increasing customer usage through loyalty programs and incentives targeting your existing customer base. Topic:
  • 27. Ch Muhammad Irfan +92-345-4426176 facebook.com/chmuhammedirfan skype id: ch.irfan786 Identify and define the major parts of a financial plan for a business: Effective financial planning involves far more than balancing your checkbook on a monthly basis. The ultimate goal of comprehensive financial planning is to gain financial clarity in all aspects of your life. How do you achieve financial clarity? By creating a ‘life plan’ that integrates your visions, values, and motivations – all unique to you and your lifestyle – with your financial goals, realities, and expectations. 1 Cash flow management When was the last time you completed a cash flow summary or a statement of financial position? To truly understand your current assets, liabilities, and net worth, it is important to identify – concretely in writing – the status of your personal and professional income and expense balance sheet. Other aspects of cash flow management include a debt elimination plan, if needed, as well as a comprehensive savings plan. 2 Investment management An in-depth, and again, regular review of your investments is another critical component of financial planning. How your portfolio is designed, the strategies you have selected, your tolerance for market risk and its effect on your portfolio returns – these should all be evaluated to ensure that they are aligned with your investment goals, your time horizon, and your expected lifestyle both today and in retirement. There are priorities and tradeoffs to be considered and investment education from a trusted professional can be of value here. 3 Tax planning In order to maximize and preserve your investment returns, an eye toward tax management is crucial. There are a number of tax-reduction strategies, methods for generating tax-efficient income, and also wealth transfer considerations – no matter your age – to understand and implement in a proactive manner. For example, corporate bonds can benefit you more when held in a tax-deferred account, while municipal bonds can better serve you in a taxable account. 4 Tax-deferred investment environments Save early and save often. Take full advantage of any employer-sponsored retirement plan that is offered to you. Whether as an employee, employer, or both, there are a number of investment plan options that can help to augment your retirement savings. Talk to your financial advisor to learn more about profit sharing plans, 401(k)s, IRAs, defined benefit plans, defined contribution plans, as well as combination plan that could best serve your needs. 5 Insurance assessment An important, and often overlooked, component of financial planning is an evaluation of the kinds of insurance you need to protect yourself, your assets, and your loved ones. Insurance types can include life, disability, umbrella, business overhead, and long-term care to name a few. Depending on your stage in life, your insurance needs (aka risk management needs) will change and evolve. 6 Business succession planning If you are a business owner, a significant portion of your financial plan will involve not only the daily operations of your business, but also the eventual transfer or sale of your business to the
  • 28. Ch Muhammad Irfan +92-345-4426176 facebook.com/chmuhammedirfan skype id: ch.irfan786 next generation or a third party. An effective and tax-efficient transition requires foresight and planning in order to maximize your benefits. 7 Revisit your written financial plan regularly Most important to the success of your financial plan? Regular monitoring, personal accountability, and course correction, as necessary, based on the unexpected curve balls that life will inevitably throw your way. At the very least, an annual review with a financial planning professional will keep you well positioned for, and informed regarding, any significant changes in your life circumstances, objectives, or needs. Topic: Write a short note of the following: Performa Cash Flow Statement: The cash flow statement is an important tool, as it explains the changes in cash and gives the information related to the business operating, investing and financing activities in a way to bring advantage to short term analysis and cash planning of the business. The basic objective of cash flow statement is to provide the information to the management about the cash receipts and the cash payments of an organization being used for efficient cash management. Since one of the important functions of a financial manager is cash management and to ensure if adequate cash is available to meet the liabilities, it is owing to the cash flow statement that the related information is derived. In this way, cash flow statement is useful to plan financial operations in an efficient manner. Cash flows are inflows and outflows of cash and cash equivalents. The cash activities are classified into three main categories of cash inflows and cash outflows. The tree categories are: - 1. Operating activities 2. Investing Activities 3. Financing Activities Break Even Analysis: A breakeven analysis is used to determine how much sales volume your business needs to start making a profit. The breakeven analysis is especially useful when you're developing a pricing strategy, either as part of a marketing plan or a business plan. To conduct a breakeven analysis, use this formula: Fixed Costs divided by (Revenue per unit - Variable costs per unit) Fixed costs are costs that must be paid whether or not any units are produced. These costs are "fixed" over a specified period of time or range of production. Variable costs are costs that vary directly with the number of products produced. For instance, the cost of the materials needed and the labor used to produce units isn't always the same. Topic: Briefly explain intellectual property and trade mark: Trademarks: Trade marks protect any symbol that indicates the source or origin of the goods or services to which it is affixed. While a trademark can be extremely valuable to its owner, the ultimate
  • 29. Ch Muhammad Irfan +92-345-4426176 facebook.com/chmuhammedirfan skype id: ch.irfan786 purpose of a trademark is to protect consumers – that is, the function of a trademark is to inform the consumer where the goods or services originate. The consumer, knowing the origin of the goods, can make purchasing decisions based on prior knowledge, reputation or marketing. Intellectual property: Intellectual property is a broad categorical description for the set of intangibles owned and legally protected by a company from outside use or implementation without consent. Intellectual property can consist of patents, trade secrets, copyrights and trademarks, or simply ideas. The concept of intellectual property relates to the fact that certain products of human intellect should be afforded the same protective rights that apply to physical property. Most developed economies have legal measures in place to protect both forms of property. Topic: Identify and define the major source of capital for your business: Often the hardest part of starting a business is raising the money to get going. The entrepreneur might have a great idea and clear idea of how to turn it into a successful business. However, if sufficient finance can't be raised, it is unlikely that the business will get off the ground. Your savings Most entrepreneurs start their companies by investing their own savings. This source of financing can be ideal – if you can afford it. It puts you in full control of how much you are going to get. Furthermore, you never have to justify yourself to investors. This last point is an important benefit. You have the freedom to operate as you see best. There is a trade-off, though: this freedom usually comes at the expense of having little money. Your credit Many entrepreneurs also use their credit – either through credit cards or a home equity loan – to start their small businesses. Be very careful about using these sources, as you could ruin your personal credit, risk your home, or both. If possible, avoid using a home equity line of credit as the risk is too high. Family members and friends One common way to finance a business is to ask friends and family members for an investment. Because this method is probably one of the riskiest ways to finance a business, we recommend against it. The problem is that if things go wrong, your friend/family relationship is affected. And in any startup business, you are guaranteed that things will go wrong at one point or another. Customers and suppliers Two great funding sources for small businesses are actually your customers and your suppliers. Customers can finance your business by prepaying for orders – or by giving you an advance. Obviously, you use these funds to handle their transaction. This funding is not easy to get, but customers may be willing to prepay if your service is unique enough or if they really need your products. Small Business Administration
  • 30. Ch Muhammad Irfan +92-345-4426176 facebook.com/chmuhammedirfan skype id: ch.irfan786 One great source of small business funding is the Small Business Administration (SBA). The SBA has a special program that offers Microloans to small business owners. Microloans can reach $50,000 (varies by state) and are much easier to get than conventional financing. They also come bundled with business training which can be very useful for first-time entrepreneurs. The SBA also works with banks to provide (a) loans. These loans are larger that Microloans and have stricter underwriting criteria. Peer-to-peer lenders Peer-to-peer (P2P) lenders offer a platform that allows people to seek financing from other individuals. The P2P platform acts as a matching service and provides basic due diligence. Like Microloans, most P2P loans are small. They usually cap at $25,000 to $35,000.  Leasing Small businesses in certain industries need equipment to get started. One way to finance the equipment is to lease it through a finance company. Most equipment leases are structured so that the finance company buys the equipment and rents it to you for a monthly payment. Once the term of the lease concludes, you can buy the equipment from the finance company for a fairly modest cost. Topic: What Is the Difference Between a Marketing & Business Plan? The business plan and the marketing plan are both essential aspects of successful businesses. These plans not only help businesses organize their operations, but help benchmark their success or failure. Since business industries change with customer demand and the economic environment, the business plan and marketing plan must evolve throughout the lifespan of the business. Business Plan: A written document describing the nature of the business, the sales and marketing strategy, and the financial background, and containing a projected profit and loss statement. A business plan is also a road map that provides directions so a business can plan its future and helps it avoid bumps in the road. The time you spend making your business plan thorough and accurate, and keeping it up-to-date, is an investment that pays big dividends in the long term. Your business plan should conform to generally accepted guidelines regarding form and content. Each section should include specific elements and address relevant questions that the people who read your plan will most likely ask. Marketing Plan: A marketing plan is a business document written for the purpose of describing the current market position of a business and its marketing strategy for the period covered by the marketing plan. Marketing plans usually cover a period of one to five years. The purpose of a marketing plan is to clearly show what steps or actions will be taken to achieve the plan goals. For example, a marketing plan might have a strategy to increase the organization's market share by ten percent. The plan would then outline the goals that need to be achieved in order to reach a ten percent increase in market share.
  • 31. Ch Muhammad Irfan +92-345-4426176 facebook.com/chmuhammedirfan skype id: ch.irfan786 Topic What are the basic principles of Time management? Here’s an overview of the seven principles of time management as outlined by Mark in the book: Have a clear vision  Ask yourself "What am I actually trying to achieve?"  The clearer you are about your vision, the more likely you are to achieve it.  It’s as much about what you’re not going to do as it is about what you are –- you are establishing limits. For example, "I will process my emails for no more than 20 minutes." Do one thing at a time  Successful people don’t take on too much. They concentrate on doing and finishing one thing at a time as far as possible.  Little and often -- the human mind works best when we apply this principle. Define your limits  The best way to be creative is not to try to think without limits, but to carefully define what those limits should be. Limitations actually encourage creativity.  If you feel you can’t get going or you're getting nowhere, it’s probably due to poorly defined limits. For instance, if you have a limited amount of time you will be able to concentrate your efforts better than if you have unlimited time. Use Closed lists  Any list that has a line drawn at the end of it can’t be added to. This enables you to deal with all the items on it without the distraction of new work being added.  Once defined, it can only stay the same or get smaller.  It doesn’t matter which order you do things, provided you are going to clear the whole list.  If you have a backlog of work to deal with, use the closed list principle. Reduced random factors These are things that stop us from completing our planned actions and can never be eliminated. Your day starts to run you rather than you running it. We tend to react to random elements; that is, we use the reactive part of our brain to react to who or whatever makes the most ‘noise’. We tend to prioritize by noise. Topic: How would you overcome the pressure on Existing Financial Resources? Overcoming Pressures on Existing Financial Resources  To overcome pressures, the entrepreneur could acquire new resources.  The acquisition of new resources is expensive, whether in terms of the equity sold or the interest payments from debt.  The need or the magnitude of the new resources required can be reduced through better management of existing resources. Managing Cash Flow
  • 32. Ch Muhammad Irfan +92-345-4426176 facebook.com/chmuhammedirfan skype id: ch.irfan786  The entrepreneur should have an up-to-date assessment of the cash position.  A daily cash sheet would provide an effective indication of any daily shortfall and of problems or errors that might have occurred.  Compare budgeted or expected cash flows with actual cash flows Managing Inventory  Perpetual inventory systems can be structured using computers or a manual system.  To check the inventory balance, it may be necessary to physically count inventory periodically.  Link the needs of a retailer with the wholesaler and producer allowing for a fast order entry and response.  Transport mode selection can also be important. Managing Fixed Assets  Generally, involve long-term commitments and large investments for the new venture.  Equipment will require servicing and insurance and affect utility costs; will also depreciate over time.  Leasing can be an alternative to buying depending on:  Terms of the lease.  Type of asset.  Usage demand.  Lease payments can be used as a tax deduction. Taxes  Withhold federal and state taxes for employees.  Pay a number of taxes (state and federal unemployment taxes and business taxes).  Allocate taxes as part of any budget.  File end-of-year returns of the business.  Consider use of a tax accountant. Topic: Enterprenurial management is distinct from Traditional management in terms of eight dimensions. Identify and explain Entrepreneur: He is a person who establishes business unit and utilizes the other factors of production like land, labor and capital. Manager: A manager is the person responsible for planning & directing the work of a group of individuals, monitoring their work, and taking corrective action when necessary. PRIMARY MOTIVES: CHARACTERISTICS ENTREPRENEUR TRADITIONAL MANAGER
  • 33. Ch Muhammad Irfan +92-345-4426176 facebook.com/chmuhammedirfan skype id: ch.irfan786 PRIMARY MOTIVES Wants freedom, goal oriented, self-reliant, and self-motivated Wants promotion and other traditional corporate rewards power motivated, TIME ORIENTATION Uses and goals of 5 to 10-year growth of the business as guides; takes action how to next step along the way Responds to quotes and budgets ; to weekly, monthly, quarterly, and annual planning horizons; and to the next promotion or transfer, TENDENCY TO ACTION Gets hand dirty; may upset employee by suddenly doing their work, Delegates action; supervising and reporting takes most energy, SKILLS Knows business intimately; more business acumen then managerial or political skills; often technically trained if in technical business; may have had profit and loss responsibility in the company, Professional management ; often business school trained; uses abstract analytical tools, people- management and political skills, ATTITUDE TOWARDS COURAGE AND DESTINY Self-confident, optimistic and courageous, Self-confident and courageous; many are cynical about the system but optimistic about their ability to outwit it. FOCUS OF ATTENTION Primarily on technology and marketplace, Primarily on events inside corporation, ATTITUDE TOWARDS RISK Likes moderate risk; invests heavily but expects to succeed, Cautions, USE OF MARKET RESEARCH Creates needs; creates products that often cannot be tested with market research; potential customers do not yet understand them; talks to Has market studies done to discover needs and guide product conceptualization,
  • 34. Ch Muhammad Irfan +92-345-4426176 facebook.com/chmuhammedirfan skype id: ch.irfan786 customers and forms own opinion, Topic: As an entrepreneur how do you minimize Patent risk in the business?  Seek a patent attorney who has expertise in your product line.  The entrepreneur should consider a design patent to protect the product design or product look.  Before making an external disclosure of an invention at a conference or to the media, or before setting up a beta site, the entrepreneur should seek legal counsel since this external disclosure may negate a subsequent patent application.  Evaluate competitor patents to gain insight into what they may be developing.  If you think your product infringes on the patent of another firm, seek legal counsel.  Verify that all employment contracts with individuals who may contribute new products  have clauses assigning those inventions or new products to the venture.  Be sure to properly mark all products granted a patent. Not having products marked  could result in loss or damages in a patent suit.  Consider licensing your patents. This can enhance the investment in a patent by creating  new market opportunities and can increase long-term revenue.